Vascular Flashcards

1
Q

GU) 25. A 30-year-old man with known Hep B surface Antigen (HBsAg), abdominal pain, malaise and weight loss is diagnosed with Polyarteritis nodosa (PAN). Which of the following is true?

A. Affects small arteries only

B. Causes aneurysms which are usually 5-10mm in size

C. Produces moth-eaten nephrograms after thrombosis of the microaneurysms

D. Angiography has signifcantly higher sensitivity than specifcity in diagnosis

E. Angiography has a much higher positive predictive value (PPV) compared with negative predictive value (NPV)

A

C. Produces moth-eaten nephrograms after thrombosis of the microaneurysms

Affects medium and small arteries.

Aneurysms are 2-3cm in size,

which when thrombosed produce moth-eaten nephrogram.

Angiography has high sensitivity and specificity

but a much higher negative than positive predictive value in diagnosis.

Diagnosis is often made with clinical features and angiography.

How well did you know this?
1
Not at all
2
3
4
5
Perfectly
2
Q

GU) 26. A 29-year-old female with new onset hypertension has normal renal tract ultrasound. Magnetic resonance Angiography (MRA) is performed which demonstrates narrowing at the distal right main renal artery. Which is the most likely diagnosis?

A. Renal artery stenosis

B. Fibromuscular dysplasia

C. Arteriosclerotic RA disease

D. Neurofbromatosis

E. Polyarteritis nodosa

A

B. Fibromuscular dysplasia

FMD accounts for 35% of RAS. It is more common in children and young adults, affecting a greater amount of females than males.

It is associated with hypertension and progressive renal insuffciency.

FMD occurs bilaterally in 2⁄3 of cases and R > L 4:1.

It usually affects the mid distal renal artery and there can be a beaded appearance of artery

How well did you know this?
1
Not at all
2
3
4
5
Perfectly
3
Q
  1. A 30-year-old male being investigated for progressive intermittent left calf claudication presents. Doppler waveforms of the popliteal artery are noted to be diminished during muscle contraction. Angiography reveals medial deviation of the popliteal artery, popliteal stenosis and post-stenotic dilatiation. Which is the most likely diagnosis?

A. Popliteal aneurysm with thrombosis

B. Ruptured Baker’s cyst

C. Popliteal artery entrapment syndrome

D. Buerger’s disease

E. Cystic adventitial disease of the popliteal artery

A

C. Popliteal artery entrapment syndrome

Stress angiography shows a normal arterial lumen with the foot in the relaxed position, with narrowing of the lumen during stress manoeuvres

How well did you know this?
1
Not at all
2
3
4
5
Perfectly
4
Q
  1. Which of the following features distinguishes coarctation from pseudocoarctation of the aorta?

A. Associated hypertension

B. Bicuspid aortic valve

C. Asymptomatic history

D. Indentation of the left lateral margin of the aortic arch in the region of the aortopulmonary window

E. Rib notching

A

E. Rib notching

The following features are not present in pseuocoarctation: rib notching, dilatation of the brachiocephalic arteries, LV enlargement and post-stenotic dilatation.

How well did you know this?
1
Not at all
2
3
4
5
Perfectly
5
Q

5) A 28-year-old male with known Marfan’s syndrome presents with chest pain and shortness of breath. An echocardiogram is performed. What are the most likely findings?

a. aortic regurgitation and dilatation

b. pulmonary stenosis

c. aortic stenosis and post-stenotic dilatation

d. global myocardial wall thickening

e. ventricular septal defect

A

a. aortic regurgitation and dilatation

Marfan’s syndrome is an autosomal dominant connective tissue disorder.

It predominantly affects the musculoskeletal system but 60–98% of patients have cardiovascular manifestations.

There is myxomatous degeneration of the aortic wall, leading to dilatation of the aortic root and aortic regurgitation.

There is an association with congenital heart disease, incomplete coarctation and atrial septal defects.

How well did you know this?
1
Not at all
2
3
4
5
Perfectly
6
Q

18) A right-sided aortic arch with mirror-image branching is most frequently associated with which congenital cardiac abnormality?

a. pulmonary atresia and ventricular septal defect

b. truncus arteriosus

c. uncomplicated ventricular septal defect

d. Fallot’s tetralogy

e. corrected transposition of the great vessels

A

d. Fallot’s tetralogy

There is a 98% incidence of associated congenital heart disease with a right-sided aortic arch with mirror-image branching.

Nearly all of these cases will be tetralogy of Fallot.

All of the given options are associated with rightsided aortic arch, as well as dextrocardia with situs inversus and double-outlet right ventricle.

Right-sided aortic arch with left subclavian artery is associated with only a 12% incidence of congenital heart disease, again with Fallot’s tetralogy being the most commonly associated abnormality

How well did you know this?
1
Not at all
2
3
4
5
Perfectly
7
Q

GU) 18) A 55-year-old, hepatitis B-positive male under investigation for painless haematuria is admitted as an emergency with unilateral loin pain and hypotension. A renal arteriogram shows multiple, bilateral, small, renal artery branch aneurysms. Which of the following antibody titres is most likely to be positive?

a. anti-double-stranded DNA

b. anti-basement membrane

c. anti-Ro

d. anti-immunoglobulin G

e. perinuclear anti-neutrophil cytoplasmic

A

e. perinuclear anti-neutrophil cytoplasmic

While several forms of arteritis can cause multiple small renal aneurysms, it is polyarteritis nodosa that does so most commonly, affecting men more than women with a mean age of 55 years, the range being 18–81 years.

It is associated with HIV and hepatitis B infection, and pANCA is usually positive.

Systemic lupus erythematosus is associated with anti-double-stranded DNA antibodies,

Goodpasture’s disease with anti-basement membrane antibodies,

Sjogren’s syndrome with anti-Ro and anti-La antibodies

and rheumatoid arthritis with antiimmunoglobulin G (anti-IgG), also known as rheumatoid factor.

How well did you know this?
1
Not at all
2
3
4
5
Perfectly
8
Q

21) A 32-year-old male presents with headaches and bilateral lower limb claudication. He is noted to have weak pulses in the lower limbs. A chest radiograph shows a ‘figure-3’ indentation of the aorta and inferior rib notching. The cardiac apex is elevated. There is no previous medical history. What is the most likely diagnosis?

a. superior vena caval obstruction

b. aortic dissection

c. coarctation of the aorta

d. aortic thrombosis

e. transposition of the great vessels

A

c. coarctation of the aorta

Adults presenting with coarctation usually have headaches (due to hypertension) and claudication (due to hypo perfusion).

The chest radiograph shows an indentation on the lateral margin of the aorta (‘figure-3’ sign) and elevation of the cardiac apex due to left ventricular hypertrophy.

Rib notching is seen in adults, usually over 20 years, and affects the third to eighth ribs.

Superior vena caval obstruction may cause rib notching but also causes a ‘nipple’ on the side of the aorta due to dilated collaterals (accessory hemizygous).

Aortic thrombosis may present similarly to coarctation, but the ‘figure-3’ sign is not seen.

Transposition presents in the first 2 weeks of life.

How well did you know this?
1
Not at all
2
3
4
5
Perfectly
9
Q

23) A 40-year-old female presents with a stroke, which is confirmed on CT of brain. She gives a history of worsening claudication of the limbs and a long history of fever and myalgia. CT of the neck and thorax shows thickening of the arterial walls of the aorta and major vessels with irregular stenotic lesions throughout the aorta, with focal areas of dilatation and stenosis in the brachiocephalic arteries and carotid arteries. What is the most likely diagnosis?

a. fibromuscular dysplasia

b. syphilis

c. atherosclerosis

d. Marfan’s syndrome

e. Takayasu’s arteritis

A

e. Takayasu’s arteritis

Takayasu’s arteritis is a granulomatous inflammatory condition of unknown cause.

It is the only aortitis to cause stenosis/occlusion of the aorta.

It produces irregular short or long stenotic lesions within the aorta.

Involvement of the great vessels usually produces multisegmental dilatation with segmental septation.

Atherosclerosis tends to occur in older patients.

Syphilis produces aneurysms of the ascending aorta with extensive calcification.

Marfan’s syndrome produces aneurysms and occasionally is associated with aortic coarctation.

How well did you know this?
1
Not at all
2
3
4
5
Perfectly
10
Q

25) A patient with known polyarteritis nodosa presents with acute left loin pain. Which of the following is most likely to be seen on ultrasound scan?

a. hydronephrosis

b. a solid mass with a perinephric collection

c. multiple renal artery aneurysms and perinephric collection

d. crossed fused ectopia

e. small kidneys with increased echogenicity

A

c. multiple renal artery aneurysms and perinephric collection

Polyarteritis nodosa is a systemic necrotizing inflammation of mediumsized and small muscular arteries.

No glomerulonephritis (small, echogenic kidneys) is present.

The condition most commonly affects the kidneys (85%) and is usually seen as multiple small intrarenal aneurysms, which may disappear due to thrombosis.

Recognized complications are perinephric or subcapsular hemorrhage due to aneurysm rupture.

Crossed fused ectopia and hydronephrosis are not recognized features of polyarteritis nodosa.

How well did you know this?
1
Not at all
2
3
4
5
Perfectly
11
Q

42) A 28-year-old male is involved in a road traffic accident and sustains chest trauma. He has chest pain and bruising over the chest with reduced blood pressure. A chest radiograph shows a shift of the trachea to the right at T3–4 level with depression of the left main bronchus and loss of clarity of the aortic knuckle. Which diagnosis should be considered?

a. bronchial rupture

b. superior vena caval laceration

c. azygos vein injury

d. aortic rupture

e. internal mammary artery injury

A

d. aortic rupture

Aortic injury is usually fatal, though some patients survive to reach hospital. A chest radiograph may show a variety of features including deviation of the trachea and esophagus (position of nasogastric tube) to the right, depression of the left main bronchus, apical pleural cap and left pleural effusion.

The presence of a mediastinal haematoma following trauma is more likely due to azygos or hemizygous vein injury or possibly internal mammary or intercostal artery injury.

Superior vena caval injury tends to cause right-sided mediastinal and lung changes.

Bronchial rupture may be accompanied by vascular injury but would tend to present with pneumomediastinum and pneumothorax with or without collapsed lung.

How well did you know this?
1
Not at all
2
3
4
5
Perfectly
12
Q

45 A 28 year old man has a Barium swallow to investigate dysphagia. The RAO views show a ‘reverse 3’ indentation in the mid oesophagus. Which of the following features would make the diagnosis of true coarctation more likely than pseudocoarctation?

(a) Ejection systolic murmur

(b) His lack of symptoms

(c) Inferior rib notching

(d) No gradient on pressure studies

(e) Widening of the superior mediastinum

A

(c) Inferior rib notching

Pseudocoarctation is an acute kink/ anterior buckling just distal to the left SCA origin. Patients are asymptomatic and there is no pressure gradient (hence collateral vessels with subsequent rib-notching are not seen). There is an ESM on auscultation and the superior mediastinum appears widened due to elongated, redundant ascending aorta and elongated aortic arch. Both true and pseudocoarctation produce the ‘figure 3’ sign on angiogram (‘reverse 3’ on RAO projections at Barium swallow) due to notching at the ligamentum arteriosum.

How well did you know this?
1
Not at all
2
3
4
5
Perfectly
13
Q

GIT) A 65-year-old man presents with a several-week history of lower abdominal pain and diarrhoea. On examination he has tenderness and guarding in the left lower quadrant. On contrast-enhanced CT, the inferior mesenteric vein is dilated, with a thin rim of enhancement around a central area of low density. What is the most likely additional pathology demonstrated on the CT?

A. sigmoid diverticulitis

B. appendicitis

C. Crohn’s disease

D. pancreatitis

E. caecal malignancy

A

A. sigmoid diverticulitis

The inferior mesenteric vein provides venous drainage for the rectum, sigmoid and descending colon, and is a potential route of spread of neoplastic and inflammatory conditions. Inferior mesenteric vein thrombosis may occur secondary to an inflammatory process, most commonly diverticulitis, or malignancy. Other potential causes include hypercoagulable states, surgery, trauma and bowel obstruction. Appearances are of an enlarged vein with rim enhancement surrounding central low density thrombus. Superior mesenteric vein thrombosis is much more common (95% of mesenteric venous thrombosis) and may follow an inflammatory or neoplastic process affecting the small intestine, caecum, and ascending and transverse colon.

How well did you know this?
1
Not at all
2
3
4
5
Perfectly
14
Q

29) A 64-year-old man presents with pain in the left arm when exercising, associated with a headache. The clinical team suspect subclavian steal syndrome. Ultrasound scan, however, shows normal flow in the carotid and vertebral arteries bilaterally. What is most likely to happen to the flow in the vessels during patient exercise to reproduce the pain?

a. reversal of flow in the right carotid artery

b. reversal of flow in the left carotid artery

c. reversal of flow in the right vertebral artery

d. reversal of flow in the left vertebral artery

e. no change

A

d. reversal of flow in the left vertebral artery

In subclavian steal syndrome (SSS), there is a stenosis in the subclavian artery proximal to the vertebral artery origin. This causes reversal of flow in the ipsilateral vertebral artery to maintain blood supply to the upper limb. If the stenosis is not severe, there is normal flow at rest, but exercise aggravates this by increasing the blood supply to the limb. As the stenosis is unable to accommodate the increased flow, the flow in the ipsilateral vertebral artery is reversed. This is termed ‘occult SSS’.

How well did you know this?
1
Not at all
2
3
4
5
Perfectly
15
Q

GIT) A 70-year-old man presents with fresh bleeding per rectum. He undergoes resuscitation, receiving 5 units of blood over the following 24 hours. Colonoscopy is unsuccessful in detecting the source of the bleeding, and he continues to pass fresh blood, although he remains haemodynamically stable. What is the most appropriate next investigation?

A. 99m Tc-labelled red blood cell radionuclide imaging

B. CT angiography

C. repeat colonoscopy

D. digital subtraction mesenteric angiography

E. abdominal ultrasound scan

A

B. CT angiography

Several imaging methods are available for use in those patients in whom endoscopy fails to detect the source of bleeding in gastrointestinal haemorrhage.

Radionuclide imaging is non-invasive and very sensitive, detecting bleeding rates as low as 0.1–0.5 ml/min. Images are acquired over several hours, enabling detection of intermittent and venous bleeding, but anatomical localization can be insensitive and variable.

Conventional angiography is invasive and requires active bleeding at the time of both imaging and contrast injection. Higher bleeding rates of 0.5–1 ml/min are required, and motion artefact from bowel peristalsis may be problematic, but it provides superior localization of the bleeding site and options for therapeutic intervention.

CT angiography is advocated as the most appropriate investigation, due to its wide availability, minimal invasiveness and high sensitivity, detecting bleeding rates as low as 0.3 ml/min in animal models. In addition, it enables assessment of a pathological lesion causing bleeding, which may be helpful in planning further management.

How well did you know this?
1
Not at all
2
3
4
5
Perfectly
16
Q

45) A 42-year-old female patient presents with a swollen calf, and deep venous thrombosis is suspected clinically. The D-dimer is elevated. Doppler ultrasound scan shows no thrombus in the thigh or calf veins. Spectral Doppler shows continuous signal with no respiratory variation. Which further investigation may be of value?

a. no further investigation – normal findings

b. pelvic ultrasound

c. CT pulmonary angiogram

d. chest radiograph

e. echocardiogram

A

b. pelvic ultrasound

Even with no clot seen, the loss of respiratory variation with continuous flow suggests a more proximal occlusion. As the limb swelling is unilateral, the most likely site of occlusion would be in the pelvic veins.

How well did you know this?
1
Not at all
2
3
4
5
Perfectly
17
Q

58) In multidetector CT angiography of the lower limbs, the effects of calcification on the images can be reduced by the use of which post-processing technique?

a. curved planar reformat

b. maximum-intensity projection

c. minimum-intensity projection

d. volume rendering

e. digital subtraction of pre- and post-contrast studies

A

a. curved planar reformat

The effects of vessel calcification can be difficult to remove from CT angiogram, and ultimately axial data may be the only method to assess the vessels accurately.

Curved planar reformat images may be helpful to show the lumen. This can be more problematic in small vessels.

Maximum-intensity projections can be severely affected by calcification, which ‘hides’ the lumen.

Digital subtraction CT is feasible but has not yet been adequately proven to be of value.

Minimum-intensity projections are of no value.

How well did you know this?
1
Not at all
2
3
4
5
Perfectly
18
Q

60) A 74-year-old male presents with low back pain. MRI shows some degenerative changes but no disc protrusion or neural compromise. A 6 cm, abdominal aortic aneurysm is seen, which has an irregular wall, with patchy high signal within mural thrombus and in the wall on T1W images. No perianeurysmal fluid to suggest leak is seen. What advice should be given regarding the aneurysm?

a. follow-up with ultrasound scan

b. follow-up with CT

c. follow-up with MRI

d. routine referral to vascular surgeon

e. emergency assessment by vascular surgeon

A

e. emergency assessment by vascular surgeon

The patchy high T1 signal in the mural thrombus and wall is suggestive of hemorrhage and the aneurysm is therefore unstable.

Impending rupture should be considered and urgent surgical assessment should be sought.

How well did you know this?
1
Not at all
2
3
4
5
Perfectly
19
Q

64) A 33-year-old female with renal failure has an indwelling right internal jugular venous catheter. She presents with swelling of the right arm. Ultrasound Doppler scan of the neck and arm veins is performed. Which feature would suggest occlusion of the right brachiocephalic vein?

a. collapse of right internal jugular vein on sniffing

b. variation of flow with respiration in right subclavian vein

c. variation with cardiac cycle in right subclavian vein

d. continuous monophasic flow in the right subclavian vein

e. collapse of the left internal jugular vein on sniffing

A

d. continuous monophasic flow in the right subclavian vein

Ultrasound evaluation of the central veins is difficult, as the brachiocephalic veins and superior vena cava cannot be directly visualized. Secondary features to confirm patency can be seen, such as collapse of the internal jugular veins on sniffing (Valsalva manoeuvre), and variability of flow with respiration and the cardiac cycle. A continuous monophasic flow with loss of variability suggests a proximal occlusion or stenosis.

How well did you know this?
1
Not at all
2
3
4
5
Perfectly
20
Q

GIT) A 45-year-old man presents with dysphagia and undergoes a double-contrast barium swallow. This demonstrates a smooth oblique indentation on the posterior wall of the oesophagus. What is the most likely cause of these appearances?

A. enlarged left atrium

B. aberrant right subclavian artery

C. aberrant left pulmonary artery

D. right-sided aortic arch

E. coarctation of the aorta

A

B. aberrant right subclavian artery

A number of anomalies of the major vessels can cause extrinsic impressions upon the oesophagus.

The commonest aortic anomaly is a right-sided aortic arch, which produces an indentation on the right lateral oesophageal wall in the absence of the normal left aortic arch impression.

An aberrant right subclavian artery originates from the aortic arch just distal to the left subclavian artery, and passes upwards and to the right, behind the oesophagus, giving rise to an oblique posterior oesophageal indentation.

In aortic coarctation, the pre- and post-stenotic dilatations of the aorta produce a characteristic reversed-3 impression upon the left wall of the oesophagus.

An enlarged left atrium andan aberrant left pulmonary artery both cause anterior indentations upon the oesophagus.

How well did you know this?
1
Not at all
2
3
4
5
Perfectly
21
Q
  1. A 16 year old with headache and hypertension has a chest radiograph which demonstrates plain radiographic signs of coarctation of the aorta. Further investigations reveal anomalous post-coarctation origin of the right subclavian artery. The ribs most likely to demonstrate inferior rib notching would be:

a. Left third to ninth ribs

b. Bilateral third to ninth ribs

c. Right third to ninth ribs

d. Bilateral first and second ribs

e. Left first and second ribs

A
  1. a. Left third to ninth ribs

Due to the anomalous origin of the right subclavian artery from the post-coarctation segment, there is no collateral flow to the intercostal arteries on the right. Subsequently, there is no right-sided rib notching.

How well did you know this?
1
Not at all
2
3
4
5
Perfectly
22
Q

@# Ped) 14. A 14 year old patient with Turner syndrome presents with severe headache. Clinical examination confirms upper limb hypertension and a murmur. Which of the following signs is likely on the plain films?

a. Boot-shaped heart

b. Snowman sign

c. Figure-of-three sign

d. Egg-on-a-string sign

e. Scimitar sign

A
  1. c. Figure-of-three sign

The above mentioned are plain radiography signs of various congenital heart diseases.

The condition described above is coarctation of the aorta.

A boot-shaped heart is a feature of tetralogy of Fallot.

Snowman sign or figureof-eight sign is seen in supracardiac TAPVD.

Scimitar sign is a feature of partial anomalous pulmonary venous return,

and egg-on-astring sign is noted in TGA.

How well did you know this?
1
Not at all
2
3
4
5
Perfectly
23
Q

96) A 35-year-old female who smokes presents with hypertension and renal impairment. Ultrasound scan shows normal appearance of the kidneys. Doppler of the renal arteries demonstrates a peak velocity of 180 cm/s and no diastolic flow. Angiography shows multiple stenosis of the renal arteries bilaterally with a normal aorta. Which therapeutic option should be offered?

a. angiotensin-converting enzyme inhibitors

b. other antihypertensive

c. angioplasty

d. surgical correction

e. no definitive therapy is of value

A

c. angioplasty

The appearances are suggestive of fibromuscular hyperplasia, which is the commonest cause of renovascular hypertension in young patients and causes 35% of all renal artery stenosis. It affects the aortic branches, but not the aorta itself. Complications include giant aneurysm and arteriovenous fistula. Angioplasty has a 90% success rate with a low restenosis rate and is the treatment of choice. Surgical correction can be performed (end-to-end anastomosis, vein grafts) but is usually reserved for refractory cases or where there is involvement of the segmental vessels rather than the main renal arteries.

How well did you know this?
1
Not at all
2
3
4
5
Perfectly
24
Q

GU) 4 A 56 year man presents with hypertension and headache. He undergoes renal investigations which show a small right kidney on US and prolonged nephrogenic phase on contrast enhanced CT. MRA shows a 50% stenosis in the right main renal artery 1 cm from the ostium. What is the most likely diagnosis?

(a) Atherosclerosis

(b) Fibromuscular dysplasia

(c) lnfrarenal aortic aneurysm

(d) Buerger’s disease

(e) Polyarteritis nodosa

A

(a) Atherosclerosis

Overall, atherosclerosis is the commonest cause of RAS. It has a particular tendency to involve the proximal 2 cm of the main renal artery in contradistinction to fibromuscular dysplasia, the seeond most common cause.

How well did you know this?
1
Not at all
2
3
4
5
Perfectly
25
Q

6 Which of the following is not an accepted indication to choose C02 angiography?

(a) EVAR stent procedure

(b) Fluoroscopic angiography where DSA unavailable

(c) IVC filter placement

(d) Previous anaphlaxis post-IVU

(e) Serum creatinine of 190 μmol/L

A

(b) Fluoroscopic angiography where DSA unavailable

The main advantages of C02-angiography is its use in patients where iodinated agents are contraindicated, e.g. renal failure or previous contrast reaction,

or for long procedures in order to limit contrast burden (e.g. EVAR placement).

C02 is not diluted by blood thus can opacify central veins more readily from a peripheral approach, which is advantageous when placing IVC filters.

The overall quality of C02 images is reduced as there is decreased density compared to Iodine, thus DSA is essential.

Additionally, multiple injections are usually required, thus radiation exposure may be increased.

How well did you know this?
1
Not at all
2
3
4
5
Perfectly
26
Q

MSK) 10 In the evaluation of a splenic injury following trauma, which of the following features favours pseudoaneurysm over active extravasation?

(a) Less apparent on delayed imaging

(b) Layering

(c) Ill-defined

(d) Increased size on delayed imaging

(e) Jet of contrast medium

A

(a) Less apparent on delayed imaging

Pseudoaneurysms are typically well-defined, rounded and may be seen to have a neck adjoining an adjacent vessel.

They are more difficult to detect on delayed images and there is no change in the haematoma.

Management may be by embolisation or surgery depending on the clinical condition of the patient.

How well did you know this?
1
Not at all
2
3
4
5
Perfectly
27
Q

Ped) 11 An infant presents within the first few weeks of life with stridor, respiratory distress, and wheezing. CXR shows left deviation of the trachea, the lateral view reveals increased density in the region of the hilum. A pulmonary sling is suspected and a CT chest is arranged for further evaluation. Which of the following findings is most likely to be seen?

(a) Anterior trachea and anterior oesophageal compression

(b) Anterior trachea and posterior oesophageal compression

(c) No compression

(d) Posterior trachea and anterior oesophageal compression

(e) Posterior trachea and posterior oesophageal compression

A

(d) Posterior trachea and anterior oesophageal compression

A ‘pulmonary sling’ refers to an aberrant left PA, which arises from the right PA and passes between the trachea and oesophagus, resulting in posterior compression of the trachea and anterior compression of the oesophagus.

It may lead to tracheo-bronchiomalacia or stenosis in up to 50% patients.

How well did you know this?
1
Not at all
2
3
4
5
Perfectly
28
Q

12- A 27-year-old man under investigation for suspected liver disease has his venous pressures measured and is found to have a hepatic venous wedge gradient of 3 mmHg. What does this indicate?

(a) There is no liver disease

(b) Pre-sinusoidal cause of portal venous hypertension

(c) Sinusoidal cause of portal venous hypertension

(d) Post-sinusoidal cause of portal venous hypertension

(e) There is Insufficient data to draw a conclusion

A

(e) There is Insufficient data to draw a conclusion

These are normal physiological measurements (1—4 mmHg), but may be seen in normal patients, extra hepatic portal hypertension and some cases of pre—sinusoidal portal hypertension.

Elevation in sinusoidal disease is seen and correlates with Child score and risk of esophageal variceal hemorrhage

How well did you know this?
1
Not at all
2
3
4
5
Perfectly
29
Q

13- When optimizing a CT study of the aorta, which of the following would not improve arterial enhancement?

(a) Increasing the injection flow rate.

(b) Increasing the iodine concentration.

(c) Increasing the duration of the injection

(d) Increasing the cardiac output

(e) Increasing the volume of the contrast medium

A

(d) Increasing the cardiac output

Cardiac output is inversely proportional to arterial opacification. The other 4 techniques are used to increase arterial opacification- Rates above 8 ml/s offer no incremental value, whilst matching dose to bodyweight will give more uniform enhancement across a patient group.

How well did you know this?
1
Not at all
2
3
4
5
Perfectly
30
Q

14- A 76-year-old man injection flow rate Iodine concentration duration of the injection cardiac output volume of contrast medium presents with central chest pain- He is hemodynamically stable and undergoes a thoracic CT study which demonstrates a hyperdense crescent in the wall of the descending thoracic aorta on the unenhanced study, which does not change following the administration of IV. contrast medium. What is the most likely diagnosis?

(a) Aortic dissection

(b) Atherosclerosis

(c) Intramural hematoma

(d) Vasculitis

(e) Mediastinal fibrosis

A

(c) Intramural hematoma

This results from the rupture of the vasa vasorum and hemorrhage in to the arterial media. It is associated with hypertension (50%, is more common in men (60%) and presents in the same manner as aortic dissection, with most (> 90%) cases non—traumatic. It has a mortality rate of around –

How well did you know this?
1
Not at all
2
3
4
5
Perfectly
31
Q

15- A 67-year-old patient with cardiac risk factors presents with left calf claudication on walking 30 yards. An MR angiogram is performed and a focal stenosis of the right popliteal artery. The vascular surgeons discuss further management. What is the best approach for subsequent catheter angiography?

(a) Antegrade, left common femoral artery

(b) Antegrade, right common femoral artery

(c) Retrograde, left common femoral artery

(d) Retrograde, left popliteal artery

(e) Retrograde, right common femoral artery

A

(b) Antegrade, right common femoral artery

The likely location and extent of the lesion is known. As intervention is likely, the antegrade (‘down the leg’) approach should be adopted on the symptomatic side. A diagnostic run should be performed to confirm disease extent as MR can over-estimate it, alliteratively there may have been disease progression depending on the elapsed time-interval

How well did you know this?
1
Not at all
2
3
4
5
Perfectly
32
Q

@# 16- Regarding congenital abnormality of the IVC. Which of the is most likely be with azygous of the IVC?

(a) Asplenia.

(b) Total anomalous pulmonary venous connection (TAPVC)

(c) Bilateral bilobed lungs

(d) Right isomerization

(e) Eparterial bronchi

A

(c) Bilateral bilobed lungs

Azygous continuation Of the IVC is congenital absence of the hepatic segment of the IVC with continuation to the right atrium occurring via the azygous or hemi-azygous veins, prevalence = 0.6%.

It is often associated with polysplenia syndrome (left isomerization) and only rarely with asplenia syndrome (right isomerization).

Polysplenia syndrome is associated with bilateral bi-lobed lungs, partial anomalous pulmonary venous connection (PAPVC), persistent left SVC, situs ambiguous, hyparterial bronchi, atrial septal defects (ASD) and double outlet right ventricle (DORV)

How well did you know this?
1
Not at all
2
3
4
5
Perfectly
33
Q

19- A 29-year-old active man presents with slowly progressive intermittent unilateral claudication brought on by exercise or standing. He has never smoked and has never family history. On examination, plantar flexion against resistance resistance reproduces the What is the likely diagnosis?

(a) atherosclerosis

(b) burger’s disease.

(c) Cystic adventitial disease of the popliteal

(d) Popliteal artery entrapment syndrome

(e) Ruptured Achilles tendon

A

(d) Popliteal artery entrapment syndrome

PAES is most common in young active male patients (60% are < 30 yrs.) and must be considered in an individual of this demographic who develops calf claudication on exercise.

It is bilateral in up to 67% and usually results from an anomalous course of the medial head of gastrocnemius which causes compression of the popliteal artery.

It can rarely be caused by a fibrous band, or muscle hypertrophy in athletes.

Surgical release of the muscle or tendon is the optimal treatment, there is no role for angioplasty/ stenting.

How well did you know this?
1
Not at all
2
3
4
5
Perfectly
34
Q

GU) 41 A 53 year old asymptomatic man is discovered to have a 1.5 cm unilateral, calcified, extrarenal solitary renal artery aneurysm. Which of the following statements is incorrect?

(a) The lesion should be treated surgically

(b) 2/3 of renal artery aneurysms are extrarenal

(c) The likeliest cause is atherosclerosis

(d) Ehlers-Danlos syndrome is a cause

(e) It is more common in women

A

(a) The lesion should be treated surgically

A small (< 2 cm) renal artery aneurysm which is well calcified may be treated conservatively.

Indications for surgery include being a woman of childbearing age (due to the increased risk of rupture in p’regnancy), interval growth, emboli to kidney or diminishing renal function.

lntrarenal aneurysms are most commonly congenital; true extra-renal aneurysms are caused by fibromuscular dysplasia, pregnancy and mesenchymal disease such as NF and Ehlers-Danlos, in addition to atherosclerosis.

How well did you know this?
1
Not at all
2
3
4
5
Perfectly
35
Q

42- Following a prolonged angioplasty procedure is noted to have a swelling at the CFA puncture site. US confirms a 4-cm pseudo aneurysm- The patient is hemodynamically stable. What is the most appropriate immediate treatment?

(a) Conservative ‘management

(b) Manual compression

(c) Stent—graft placement

(d) Surgical repair

(e) US-guided thrombin injection

A

(e) US-guided thrombin injection

Femoral artery pseudoaneurysms occur in 0.2% of diagnostic and 8% of interventional procedures; compression of adjacent structures or rupture are potential complications.

Risk factors include 10’” puncture, larger catheter size, lengthy procedures, and peri-procedural anti- coagulation-

Treatment can be conservative for small aneurysms < 2 cm.

US-guided thrombin injection has replaced US-guided compression as the treatment of choice in uncomplicated cases due to greater success rate, reduced procedural time and better patient tolerance-

Stentgrafts are more commonly used in larger diameter arteries and have a lower complication rate compared to surgery.

Surgery is reserved for cases of rapid aneurysm expansion, rupture, infection, or failure of other treatments.

How well did you know this?
1
Not at all
2
3
4
5
Perfectly
36
Q

49- A 53-year-old man presents with abdominal pain. A post- contrast injection CT examination is performed in the portal phase which reveals an enlarged aorta ‘With an eccentric thrombosed channel, extending over 8 cm. Which of the following features makes a thrombosed aneurysm more likely than a chronic dissection?

(a) Involvement of the SMA

(b) Small aortic lumen

(c) Mural calcification displaced outwards

(d) Thrombus extending over 6 cm cranio-caudally

(e) Total aortic diameter > 5 cm

A

(c) Mural calcification displaced outwards

A thrombosed aneurysm and thrombosed blood within the false lumen of a chronic dissection can have similar appearances.

In thrombosed aneurysm, the calcium is typically displaced outwards, thus appears to be pushed outside from the aortic lumen (in dissection any calcium, if present would be displaced medially, ‘inside’ the lumen),

in addition there is likely to be extensive calcification seen elsewhere in the wall.

Dissection usually extends over a longer (> 6 cm) cranio-caudal distance and, although the overall aortic size is normal in the acute phase, it can be very large (> 5 cm) in the chronic phase.

The aortic lumen is likely to be normal or small in dissection, but is expected to be large with an aneurysm.

Involved vessels are not a reliable guide, but an aneurysm tends to involve the lumbar branches, a dissection of the renal arteries and the SMA.

How well did you know this?
1
Not at all
2
3
4
5
Perfectly
37
Q

Pd) 49 A 10 year old boy presents to his GP with back-pain. He is found to have a scoliosis, but is also noted to be tall for his age and rather thin. Which of the following supports the diagnosis of Marfan’s syndrome over homocystinuria?

(a) Aortic aneurysm

(b) Lens dislocation

(c) Mental retardation

(d) Osteoporosis

(e) History of pulmonary emboli

A

(a) Aortic aneurysm

The conditions have a similar phenotype. In Marfan’s there is MV prolapse, aortic regurgitation, coarctation, and dissecting aneurysm of the ascending aorta; lens dislocation is upwards and outwards (down/ inward in homocystinuria).

Homocystinuria is an enzyme deficiency disorder, leading to homocystine build-up which causes a defect in collagen structure.

Features which help distinguish it from Marfan’s include the propensity for thrombo-emboli disease and osteoporosis (osteopaenia in Marfan’s).

How well did you know this?
1
Not at all
2
3
4
5
Perfectly
38
Q

GIT) 19 A patient is referred for chemo-embolisation of a hepatocellular carcinoma in the right lobe of the liver. The initial angiogram demonstrates that the lesion is supplied from the superior mesenteric artery. What proportion of patients have an arterial supply to liver from the SMA?

(a) 7%

(b) 14 %

(c) 17 %

(d) 21 %

(e) 25 %

A

(e) 25 %

14% of patients have an accessory right hepatic artery, 7% have a replaced right hepatic artery and 4% have a totally replaced hepatic artery arising from the SMA.

How well did you know this?
1
Not at all
2
3
4
5
Perfectly
39
Q

Ped) 20 A 4 year old boy presents following a recent history of a fever, cervical lymphadenopathy, and a rash around both elbows and conjunctivitis. What is the most likely diagnosis?

(a) Fibromuscular dysplasia

(b) Giant cell arteritis

(c) Henoch Schonlein purpura

(d) Kawasaki’s disease

(e) Takayasu’s disease

A

(d) Kawasaki’s disease

Kawaski’s disease is an idiopathic acute febrile, multisystem vasculitis.

It is usually self-limiting, with peak presentation at 1-2 years.

Complications include coronary artery aneurysms, myocarditis, and acute Ml.

Treatment is with aspirin and gamma globulins.

How well did you know this?
1
Not at all
2
3
4
5
Perfectly
40
Q

23 A 53 year old man presents with right calf swelling. He had a previous right leg DVT treated 5 years previously at another hospital and describes the current symptoms as being similar. US shows echoic material extending 5 cm within the mid right SFV. What feature most strongly supports a diagnosis of acute DVT over chronic changes from his previous DVT?

(a) Enlarged vein diameter

(b) Hyperechoic material in the vein

(c) Non-corrlpressible vein

(d) Presence of collateral veins

(e) Vein wall thickening

A

(a) Enlarged vein diameter

It may be difficult to distinguish acute from chronic DVT, and previous imaging if available can be helpful. Signs more in keeping with chronic DVT include wall thickening, collateral vessels, and echogenic material of the same or increased echoreflectivity as compared to surroundingtissue; vein diameter is often reduced.

Acute thrombus is of lucent-tointermediate echogenicity as compared to surrounding tissues, is seen to expand the vein, is often ‘free-floating’ within the lumen in the first week, and may be compressible in the early stages.

How well did you know this?
1
Not at all
2
3
4
5
Perfectly
41
Q

GU) 25 A 30 year old woman presents with a history of hypertension and headache. She undergoes renal angiography which demonstrates alternating areas of stenosis and ectasia in the distal third of both main renal arteries. What is the likeliest diagnosis?

(a) Atherosclerosis

(b) Takayasu’s arteritis

(c) Polyarteritis nodosa

(d) Fibromuscular dysplasia

(e) Buerger’s disease

A

(d) Fibromuscular dysplasia

Overall, the commonest cause of renal artery stenosis is atherosclerosis, but FMD accounts for up to 30% of all cases, and is commoner in young patients. It typically affects the mid and distal portion of the artery and a ‘string of beads’ appearance is often seen

How well did you know this?
1
Not at all
2
3
4
5
Perfectly
42
Q

27 A 68 year old man presents with a lower GI haemorrhage. He is fluid resuscitated and an angiogram is performed to localise the site of bleeding. Initial selective angiogram of the IMA shows a distal bleeding point within the pelvis. Which of the following arteries is the more likely source of the bleed?

(a) Median sacral

(b) Middle rectal

(c) Sigmoid

(d) Inferior epigastric

(e) Internal pudendal

A

(c) Sigmoid

Branches of the IMA include left colic, sigmoidal branches, superior rectal, and marginal arteries. Medial sacral comes directly off the aorta; middle rectal and internal pudendal arteries are branches of the IIA; the inferior epigastric arteries are branches of the EIA.

How well did you know this?
1
Not at all
2
3
4
5
Perfectly
43
Q

44 Which of the following conditions is not associated with Raynaud’s phenomenon?

(a) Atherosclerosis

(b) Rheumatoid arthritis

(c) Sarcoid

(d) Takayasu disease

(e) Trauma

A

(c) Sarcoid

Raynaud’s disease is the primary form of vasospasm induced digital ischaemia following cold or emotional stimuli. The colour changes typically seen are pallor, cyanosis, then eventually redness with reperfusion. Raynaud’s phenomenon is secondary vasospasm with obstruction, causes include atherosclerosis (most frequent; either in situ or due to emboli), arterial trauma, Buerger’s, Takayasu’s, drugs (ergot), and collagen vascular diseases (RhA, scleroderma, SLE). Radiological investigations include angiograms performed at ambient temperatures, followed by ‘stress angiogram’ performed after immersing the hand in ice water for 20 seconds. Sarcoidosis is not associated.

How well did you know this?
1
Not at all
2
3
4
5
Perfectly
44
Q

52 A 29 year old women presents with pain in her left forearm. She has previous been investigated for hypertension, arthralgias, and weight loss. The arm BPs are unequal. Aortic angiogram reveals long-segment smooth stenoses at the origin of the left subclavian artery, within the abdominal aorta and in the right renal artery. What is the most likely diagnosis?

(a) Buerger’s disease

(b) Giant cell arteritis

(c) Polyarteritis nodosum

(d) Syphilitic aortitis

(e) Takayasu disease

A

(e) Takayasu disease

Takayasu (‘pulseless’) disease is a granulomatous inflammation of unknown aetiology affecting mainly elastic arteries, i.e. aorta and its major branches and the pulmonary arteries causing stenoses and occasionally aneurysms.

It is more common in female patients (x 8) and presents at a young age (90% < 30 years).

In up to 50% the early phase is marked by systemic symptoms such as fever, weight loss, night sweats, arthralgia; mean time from disease development to diagnosis is 8 years.

How well did you know this?
1
Not at all
2
3
4
5
Perfectly
45
Q

Ped) 69 What is the most common cause of renal artery stenosis in a child?

(a) Fibromuscular dysplasia

(b) Middle aortic syndrome

(c) Neurofibromatosis

(d) Takayasu’s arteritis

(e) William’s syndrome

A

(a) Fibromuscular dysplasia

All are causes of RAS, other causes include polyarteritis nodosa, postrenal transplantation and post-radiotherapy. FMD is the most common, the other conditions are rare. In adults, the majority of cases of RAS is secondary to atherosclerosis.

How well did you know this?
1
Not at all
2
3
4
5
Perfectly
46
Q

75 A 63 year old man previously treated for lung cancer presents with a gradual onset of dyspnoea, headache and facial swelling with prominent veins over the preceding weeks. A CT is requested.

Which of the following collateral veins are least likely to be enlarged?

(a) Azygous vein

(b) Hemiazygous veins

(c) Internal thoracic veins

(d) Lateral thoracic veins

(e) Pericardial Veins

A

(e) Pericardial Veins

The extent and location of collateral opacification in SVCO at CT depends on the location/ degree of obstruction and the site/ timing of contrast injectiop.

Major collateral pathways include the azygous, hemiazygous, internal thoracic, lateral thoracic veins, and the thoraco-acromioclavicular venous complex.

Minor collateral pathways include the anterior jugular, the pericardia! and umbilical veins; the parascapular, pericardia! superficial, thoracoabdominal and epigastric veins may also be opacified.

How well did you know this?
1
Not at all
2
3
4
5
Perfectly
47
Q

GU) 17. A 50-year-old male presents with history of weight loss, hypertension and headaches. Bloods show leucocytosis, eosinophilia and raised ESR. A selective renal angiogram shows bilateral, multiple small vessel aneurysms. Te renal arteries are normal. What is the most likely diagnosis?

(a) Rheumatoid disease

(b) Polyarteritis nodosa

(c) Systemic lupus erythematosus

(d) Intravenous drug abuse

(e) Atrial myxoma

A

(b) Polyarteritis nodosa

Appearances are virtually diagnostic of polyarteritis nodosa. The condition is characterised by focal areas of necrotising arteritis with fbrinoid necrosis and small aneurysm formation in skin, kidneys, cardiovascular system and central nervous system.

48
Q
  1. A 34 year old IV drug abuser presents with fever, rigors and back pain. Blood cultures reveal staphylococcal septicaemia. CT demonstrates a mycotic aneurysm. Which of the following is the most likely CT feature?

a. Fusiform shape

b. Perianeurysmal soft-tissue mass

c. Pseudoaneurysm

d. Periaortic gas collection

e. Mural thrombus

A
  1. b. Perianeurysmal soft-tissue mass

Mycotic aneurysms are usually saccular true aneurysms.

Periaortic soft-tissue mass is a common feature seen in up to 48% of cases.

Periaortic gas is an uncommon feature.

Mural thrombus and calcification are rare features.

49
Q
  1. A 70 year old man undergoes surgery for AAA. Two weeks following surgery, he is readmitted to the A&E department with abdominal pain and fever. Palpation of the abdomen suggests a pulsatile mass. A CT angiogram is performed, which does not demonstrate contrast extravasation. Which of the features on CT angiogram would be most worrisome?

a. Presence of a pseudoaneurysm

b. Periaortic soft tissue

c. Thickening of a fluid-filled third part of the duodenum

d. Some ectopic gas in the vicinity

e. Loss of fat plane between the grafted aorta and the adjacent duodenum

A
  1. c. Thickening of a fluid-filled third part of the duodenum

Two weeks post-procedure, all the other features including the presence of ectopic gas may be postoperative.

However, presence of a thickened fluid-filled bowel loop would be extremely worrying for an aorto-enteric fistula.

Presence of ectopic gas beyond four weeks is much more likely to be abnormal.

50
Q
  1. A seven year old boy with no known medical history presents with hypertension and postprandial abdominal pain. CT reveals an abnormality in the abdominal vasculature. Subsequent angiogram demonstrates occlusion of the coeliac axis and superior mesenteric artery and tapering of the mid-aorta. Delayed imaging shows vessel reconstitution through collaterals. The most likely diagnosis is?

a. Takayasu arteritis

b. Midaortic syndrome

c. Neurofibromatosis

d. Marfan’s syndrome

e. Syphilitic aortitis

A
  1. b. Midaortic syndrome

Midaortic syndrome is a rare vascular abnormality which manifests as narrowing of the abdominal aorta and its branches (including renal arteries – hence hypertension).

Its cause is unknown but it is noninflammatory and nonatheromatous.

Patients typically present after the age of five years.

Diagnosis is made with angiography which reveals smooth, segmental stenoses.

Takayasu arteritis (TA) and neurofibromatosis (NF) are amongst the differentials (the latter can have midabdominal aortic stenosis).

However, TA usually affects older patients (1266 years) and affects females to a much greater degree (M: F¼1:8).

Patients with NF and Marfan’s would usually have prior medical history.

Syphilitic aortitis is rare and typically affects older patients (40–65 year olds).

51
Q
  1. A 60 year old hypertensive man presents with sudden-onset severe inter-scapular pain. CT reveals a high attenuation crescent in the wall of the proximal ascending aorta with inward displacement of part of the calcific wall, which extends just up to the brachiocephalic artery. Post-contrast scan does not demonstrate leak of contrast into the crescentic area or mediastinum. The patient is hemodynamically stable. The next appropriate course of action is:

a. Urgent cardiothoracic referral

b. Contact the interventional radiologist for consideration of endovascular stenting

c. Follow-up scan after 24 hours

d. Medical treatment aimed at controlling blood pressure

e. Emergency cardiac MRI

A
  1. a. Urgent cardiothoracic referral

The features described are that of an acute type A intramural haematoma which needs to be treated similarly to that of a Stanford type A aortic dissection and so cardiothoracic surgical opinion is warranted. CT has delineated the extent of the acute intramural haematoma and further imaging would add little. These patients are at increased risk of progressing to a true dissection which carries a high mortality. Whilst endovascular treatment may be considered for selective cases, involvement of the proximal ascending aorta will preclude this.

52
Q
  1. A 30 year old female with uncontrolled hypertension undergoes an MR angiogram of the renal arteries. This reveals bilateral renal artery abnormalities. The most likely abnormality is:

a. Bilateral ostial stenosis

b. Bilateral long segment stenosis

c. Intrarenal aneurysms

d. Atretic renal arteries with extensive collateralisation

e. Multiple stenoses of the mid portion of the renal arteries

A
  1. e. Multiple stenoses of the mid portion of the renal arteries

In a female patient of this age, fibromuscular dysplasia is the most likely abnormality.

Fibromuscular dysplasia involves the mid and distal portions of the renal artery as well as the intrarenal branches, with multiple stenoses and aneurysms revealing a string-of-beads appearance.

The ostia and the proximal portion are much less commonly involved.

It responds very well to angioplasty, unlike ostial atherosclerotic disease which often needs stenting. Typically it is the medial layer that is affected although all layers can demonstrate changes.

53
Q
  1. Following a deceleration injury in a road traffic accident, a young man presents to the A&E department with shock, chest wall contusion and severe chest pain. An aortic injury is suspected. Which of the following is least likely?

a. A normal chest radiograph has high negative predictive value

b. Aortic injury is the usual cause of mediastinal haematoma

c. The aorta just beyond the left subclavian artery is the most common site of injury

d. Aortic rupture is usually circumferential

e. A non-contrast CT scan has a high negative predictive value in the absence of demonstrable mediastinal haematoma

A
  1. b. Aortic injury is the usual cause of mediastinal haematoma

A normal PA chest radiograph has a very high negative predictive value (95–98%) and the absence of a mediastinal haematoma on a non-contrast CT also effectively rules out the presence of aortic injury.

The source of the mediastinal haematoma is usually the azygous, hemiazygos, internal mammary and intercostal vessels.

Aortic injury is usually circumferential (85%).

54
Q

QUESTION 10 A young woman with Turner’s syndrome is found to be hypertensive. On examination, her femoral pulses are delayed, relative to the carotid pulses. In addition there is a mid to late systolic murmur. Which one of the following is the most likely radiological finding?

A An ‘8’ sign due to modelling deformities of the major thoracic vessels

B An enlarged external mammary artery on a lateral plain chest radiograph

C Elevated left ventricular apex

D Rib notching affecting all ribs

E Superior rib notching

A

C Elevated left ventricular apex

Coarctation of the aorta is associated with inferior rib notching (which takes several years to develop) and the resultant hypertension often produces left ventricular hypertrophy.

The first two ribs are generally spared as the intercostal arteries are supplied via the cost cervical trunk proximal to the coarctation and therefore do not contribute to the collateral circulation.

55
Q

Ped) QUESTION 30 A 5-year-old boy who had a coarctation of his aorta repaired 12 months ago requires follow-up. Which of the following imaging modalities is the gold standard?

A Conventional angiography

B CT

C Echocardiogram

D MRI

E Plain radiograph

A

D MRI

An echocardiogram can be used but it becomes more difficult to assess the coarctation repair as the child gets older.

56
Q
  1. A 60-year-old man presents with a history of headache, vertigo, ataxia, and intermittent pain and weakness in his left arm initiated by using the left arm for daily activities. On examination, the left radial pulse is weak and the systolic blood pressure on the left side is reduced by 30 mmHg. Doppler ultrasound reveals reversal of flow in the left vertebral artery. What is the likely underlying pathology?

A. Critical stenosis of right middle cerebral artery (MCA).

B. Critical stenosis of left MCA.

C. Critical stenosis of third part of left subclavian artery.

D. Critical stenosis of left vertebral artery.

E. Critical stenosis of the origin of left subclavian artery.

A
  1. E. Critical stenosis of the origin of the left subclavian artery.

The vertebral artery arises from the first part of the subclavian artery. Stenosis/obstruction of the subclavian artery at its origin (i.e. proximal to the origin of the vertebral artery) leads to reversal of flow in the vertebral artery to maintain circulation to the ipsilateral upper limb. This effectively ‘steals’ blood from the posterior cerebral circulation, resulting in symptoms of vertebro-basilar insufficiency. Exercising the ipsilateral extremity triggers the symptoms of vertebro-basilar and/or brachial insufficiency.

Atherosclerosis is the most common (94%) acquired cause of subclavian steal syndrome. It is more common in males and on the left side, with a ratio of 3:1.

Additional lesions of extracranial arteries are often seen (up to 81%) when symptoms of vertebro-basilar insufficiency are present. However, the underlying pathology for this syndrome is stenosis/occlusion of the origin of the subclavian artery.

Reversal of vertebral artery flow can be demonstrated by colour Doppler imaging, augmented by arm exercise/blood pressure cuff inflation above systolic pressure for 5 minutes.

Aortic arch angiography/MR angiography may be performed to demonstrate the subclavian stenosis/ occlusion. Percutaneous angioplasty and surgical bypass are the treatment options.

57
Q

Ped) 11. A 4-month-old boy is having a barium swallow done in your department for suspected reflux. There is no relevant past medical history. On the AP view you notice a filling defect in the mid oesophagus. Your consultant recommends doing a lateral swallow to further assess this. This reveals a focal area of compression of the oesophagus anteriorly in the mid oesophagus. On the screening images you also notice a posterior impression on the trachea at this level. What is the most likely cause for this finding?

A. Duplication cyst.

B. Double aortic arch.

C. Lymphadenopathy.

D. Aberrant right subclavian artery.

E. Aberrant left pulmonary artery

A
  1. E. Aberrant left pulmonary artery.

Double aortic arch compresses the trachea anteriorly and the oesophagus posteriorly.

Aberrant right subclavian (with a left-sided aortic arch) causes posterior indentation of the oesophagus, but no indentation of the trachea.

An identical appearance is seen on the lateral view with a right-sided aortic arch and an aberrant left subclavian.

Aberrant left pulmonary artery commonly presents any time up to early childhood, with stridor, wheezing, recurrent infections, and feeding problems being amongst the most common symptoms. It can be associated with a PDA or ASD.

58
Q

GU) 17. A 40-year-old female with uterine fibroids is referred for uterine artery embolization. Which of the following statements regarding the relevant arterial anatomy is incorrect?

A. Uterine artery is the first or second branch of the anterior division of internal iliac artery in 51% of cases.

B. The ipsilateral ovarian artery often replaces an absent uterine artery.

C. The ovarian artery has a characteristic corkscrew appearance on angiogram.

D. Utero-ovarian anastomosis is identified in less than 5% of cases.

E. Ovarian artery supply to fibroids is more frequently found in women with a history of previous pelvic surgery

A
  1. D. Utero-ovarian anastomosis is identified in less than 5% of cases.

Uterine arteries are the predominant source of blood supply to the fibroids in most cases.

There is considerable variation in the pelvic arterial anatomy, a thorough knowledge of which is essential to carry out uterine artery embolization safely and effectively.

The internal iliac artery bifurcates into anterior and posterior divisions in 57–77% of the general population.

The uterine artery arises as the first or second branch of the anterior division of internal iliac artery in 51% of cases.

The internal iliac artery trifurcates into uterine artery, the anterior, and the posterior divisions in 15–40% of the general population.

The uterine artery arises as the first branch of the internal iliac artery in 6% of cases.

The uterine artery may be replaced by small branches or may be absent.

The ipsilateral ovarian artery often replaces an absent uterine artery.

Ovarian arterial supply to fibroids is more common in those with previous history of pelvic surgery or tubo-ovarian disease and/or large fundal fibroids. The extent of ovarian supply should be assessed with flush pelvic aortography, preferably after uterine artery embolization.

The ovarian artery arises from the aorta just below the renal arteries in 80–90% of cases and has a characteristic corkscrew appearance.

Normal ovarian arteries are not usually identified on angiography because of their small calibre.

Utero-ovarian anastomosis is seen in 10–30% of cases and left-to-right uterine anastomosis in 10% of cases.

59
Q
  1. A 45-year-old woman presents with significant ongoing melaena, tachycardia and hypotension. Multidetector CT mesenteric angiography (MDCTA) is requested prior to consideration of mesenteric embolization. Which of the following statements regarding gastrointestinal (GI) bleeding and MDCTA is true?

A. Oral contrast should be administered to identify the causative lesion.

B. Scans are usually performed in the arterial phase only from diaphragm to ischial tuberosity.

C. Acute GI bleeding can be intermittent. Failure to demonstrate active bleeding does not prove cessation of bleeding.

D. Suture material, dense foreign bodies or faecolith can be easily distinguished from contrast extravasation.

E. The lowest detectable bleeding rate with MDCTA is 2 ml/min.

A
  1. C. Acute GI bleeding can be intermittent and failure to demonstrate active bleeding does not prove cessation of bleeding.

Acute GI bleeding is a medical emergency associated with high mortality and morbidity, especially in those presenting with hemodynamic instability. The majority (75%) of GI bleeds cease spontaneously, but it can recur in 25% of cases. Accurate early diagnosis of the bleeding source is crucial. Endoscopy is commonly used to identify and treat the source of bleeding. However, it is of limited value in massive haemorrhage (due to difficulty in visualizing the bleeding point) and in assessing the distal duodenum and most of the small bowel.

MDCTA is a rapid and accurate diagnostic method used to identify the site and in some cases the cause of active GI bleeding. However, acute GI bleeds (even when massive) are intermittent and failure to demonstrate active bleeding does not imply cessation of bleeding. The reported lowest detectable rate of bleeding with MDCTA, in animal and in vitro studies, is 0.35ml/min.

Suture material, clips, foreign bodies, and faecoliths may lead to false–positive results. An unenhanced scan is therefore essential. A portal venous phase scan is useful in identifying venous bleeds, therefore the protocol should include unenhanced and post-intravenous contrast arterial and portal venous phase scans.

60
Q

Ped) 20. A cardiac MRI is being carried out on an infant for a conotruncal rotational abnormality. It is clear that this infant has 150 clockwise rotation of the great vessels. What conotruncal rotation abnormality does this infant have?

A. Normal rotation.

B. Situs inversus.

C. L-transposition.

D. D-transposition.

E. Double-outlet right ventricle.

A
  1. B. Situs inversus.

The primitive truncus is a midline structure that persists to a degree in truncus arteriosus abnormality. During normal development the truncus divides into the aortic and pulmonary trunks, which then undergo 150° anticlockwise rotation.

In situs inversus there is 150° clockwise rotation.

Transposition of the great arteries (TGA) is characterized by 30° rotation,

anticlockwise in D-TGA

and clockwise in L-TGA.

Double-outlet right ventricle displays 90° anticlockwise rotation.

61
Q

@# 28. A 55-year-old man with a recent diagnosis of multifocal hepatocellular carcinoma is referred for transarterial chemoembolisation. Which of the following statements regarding hepatic arterial anatomy is true?

A. The classic hepatic arterial anatomy, with the proper and hepatic artery dividing into the right and left hepatic arteries, is seen in approximately 80% of the population.

B. Accessory left hepatic artery from left gastric artery is seen in 25% of cases.

C. Replaced right hepatic artery commonly arises from the gastroduodenal artery.

D. Replaced left hepatic artery commonly arises from the left gastric artery.

E. The common hepatic artery is a branch of the superior mesenteric artery.

A
  1. D. Replaced left hepatic artery commonly arises from the left gastric artery.

The classic hepatic arterial anatomy, with the proper and hepatic artery dividing into the right and left hepatic arteries, is seen in approximately 55% of the population.

Variations in hepatic arterial anatomy are common.

A replaced right hepatic artery from the superior mesenteric artery is seen in 11%.

A replaced left hepatic artery arising from the left gastric artery is seen in 10%.

The entire hepatic trunk may be replaced, which may arise from the superior mesenteric artery (4.5%) or left gastric artery (0.5%).

The common hepatic artery is a branch of the coeliac axis.

62
Q

GIT) 28. A 65-year-old male with a pancreatic head mass and obstructive jaundice undergoes percutaneous cholangiogram and external biliary drain insertion via the right lobe of the liver. The patient returns for a biliary stent insertion. On removing the external drain there is significant arterial bleed from the puncture site. A selective coeliac axis angiogram does not revealany abnormality, but pulsatile bleeding persists. What would you do next?

A. Selective left gastric angiogram.

B. Selective superior mesenteric angiogram.

C. Selective inferior mesenteric angiogram.

D. Selective gastroduodenal artery angiogram.

E. Embolise coeliac axis.

A
  1. B. Selective superior mesenteric angiogram.

Variations in hepatic arterial anatomy are common.

According to Michel classification, the classic hepatic arterial anatomy with the hepatic artery proper dividing into the right and left hepatic arteries is seen in only 55% of the population.

A replaced right hepatic artery from the SMA is seen in 11% and an accessory right hepatic artery from the SMA is seen in 7% of the population.

A selective SMA angiogram should therefore be performed in this case.

A selective left gastric angiogram is not required as it is a branch of the coeliac axis.

The inferior mesenteric artery does not supply the liver.

The coeliac axis divides into the common hepatic, left gastric, and splenic arteries. Embolizationof the coeliac axis is therefore not an option.

63
Q
  1. A 60-year-old male awaiting cardiac bypass surgery undergoes Doppler assessment of leg veins to check suitability for a vein graft. On ultrasound, incidental note is made of 1.8-cm popliteal artery aneurysm with mural thrombus. Which of the following statements regarding popliteal artery aneurysm is false?

A. It is bilateral in 50–70% of cases.

B. It is associated with abdominal aortic aneurysm in 30–50% of cases.

C. Symptomatic patients present with effects of distal embolization.

D. It may be missed on conventional angiography.

E. It should be treated only when symptomatic.

A
  1. E. It should be treated only when symptomatic.

Popliteal artery aneurysm (>0.7 cm in diameter) is the most common peripheral artery aneurysm. It is commonly associated with aneurysms in other locations—abdominal aortic aneurysm in 30–50% and contralateral popliteal aneurysm in 50–70% of cases.

They are more common in men (10:1 to 30:1) in their sixth and seventh decades. It is important to diagnose popliteal artery aneurysms due to significant associated risk of limb-threatening thrombo-embolic complications. Due to the high risk of complications, it is recommended that popliteal aneurysms should be repaired regardless of the symptoms or size, unless the patient is high risk for surgery due to associated co-morbidity.

64
Q
  1. A 25-year-old baseball player presents with a history of worsening pain, diffuse oedema and discoloration of the right upper limb following a game. Doppler ultrasound demonstrates occlusion of the axillary and subclavian veins. He undergoes catheter-directed thrombolysis successfully. Check venogram demonstrates external compression from scalenus muscle. What is the diagnosis?

A. May–Thurner syndrome.

B. Nutcracker syndrome.

C. Paget–Schroetter syndrome.

D. Trousseau syndrome.

E. Virchow syndrome.

A
  1. C. Paget–Schroetter syndrome.

This is also known as ‘effort’ syndrome and is the name given to thrombosis of the axillary and subclavian veins usually due to anatomic compression in the costoclavicular space of the thoracic outlet. It is commonly seen in young healthy adults who are involved in activities with repetitive shoulder–arm movements.

Doppler ultrasound and MR venogram are useful in diagnosis. Pharmaco-mechanical or catheter-directed thrombolysis, followed by surgical decompression of the thoracic outlet is recommended for optimal treatment.

65
Q

MSK) 60. A 25-year-old marathon runner presents with a history of right calf pain during exercise. Popliteal artery entrapment is suspected clinically. Which of the following statements regarding imaging of popliteal artery entrapment syndrome (PAES) is true?

A. In PAES, the popliteal artery is compressed with the ankle in the neutral position.

B. A normal Doppler ultrasound of the popliteal artery excludes the diagnosis.

C. In the normal popliteal fossa, the popliteal artery and vein pass lateral to the medial head of the gastrocnemius and are surrounded by fat.

D. The anatomical abnormality is invariably unilateral.

E. Catheter angiography is the gold standard for the diagnosis.

A
  1. C. In the normal popliteal fossa, the popliteal artery and vein pass lateral to the medial head of the gastrocnemius and are surrounded by fat.

Popliteal artery entrapment syndrome is a developmental abnormality resulting from an abnormal relationship between the popliteal artery and neighboring muscles.

It is commonly seen in healthy young adults and can present with symptoms of intermittent claudication or thromboembolism.

The anatomical abnormality occurs bilaterally in 27–67%. The popliteal vessels normally pass lateral to the medial head of gastrocnemius. An anomalous origin of the medial head or an anomalous course of the popliteal artery may result in extrinsic compression of the artery.

Doppler and digital subtraction angiography (DSA) findings may be non-specific with a wide spectrum of findings. A normal Doppler or DSA with neutral ankle position does not exclude the diagnosis.

Provocative measures with ankle dorsiflexion and plantar flexion may be useful in confirming the diagnosis on Doppler and DSA, but they do not demonstrate the underlying anatomical cause.

Non-invasive assessment with CT angiogram or MR angiogram is preferred as they also demonstrate the anatomical abnormality.

66
Q
  1. A 30-year-old male mechanic presents with digital ischaemia. Catheter angiogram demonstrates occlusion of the distal ulnar artery and abrupt occlusion of some of the digital arteries. The radial artery is patent and there is filling of the superficial palmar arch via deep palmar arch collaterals. What is the diagnosis?

A. Hypothenar hammer syndrome.

B. Peripheral embolic disease.

C. Raynaud’s disease.

D. Thoracic outlet syndrome.

E. Takayasu arteritis.

A
  1. A. Hypothenar hammer syndrome.

This is a post-traumatic vascular insufficiency of the hand. Any form of repetitive blunt trauma to the hypothenar eminence may result in intimal injury to the terminal ulnar artery or proximal superficial palmar arch, leading to thrombotic occlusion, aneurysms, and distal thromboembolism.

The symptoms may be similar to Raynaud phenomenon or other causes of embolic occlusion, but the location of the abnormality on imaging is specific for hypothenar hammer syndrome.

Angiography may demonstrate a ‘corkscrew’ appearance in the symptomatic and in the asymptomatic contralateral hand. Distal occlusion due to embolic phenomenon is seen in 50% of cases.

Takayasu arteritis affects the proximal aortic branch vessels.

67
Q
  1. A 70-year-old man presents with a 6-month history of cramping pain in the left calf brought on by walking and settling with rest. For the past 3 weeks he has been experiencing pain at rest, which is relieved by dependency of the foot. On examination, the popliteal and tibial pulses are absent. There is no ulceration or gangrene. What is the diagnosis?

A. Intermittent claudication.

B. Critical limb ischaemia.

C. Acute limb ischaemia.

D. Nerve root compression.

E. Diabetic neuropathy.

A
  1. B. Critical limb ischaemia.

Muscle pain or discomfort in the lower limb brought on by exercise and relieved by rest within 10 minutes is termed intermittent claudication. This is secondary to reduced perfusion, which may be enough during periods of rest, but on exercise is insufficient to meet the metabolic demand of the muscles.

With progression of disease, these symptoms may be noticed at rest. Typical ischaemic rest pain is relieved by dependency of the foot secondary to gravity-aided blood flow. Critical limb ischaemia is the presence of rest pain or tissue loss with ulcers or gangrene. This should be distinguished from acute limb ischaemia. The term ‘critical limb ischaemia’ implies chronicity with symptoms being present for at least 2 weeks.

Diabetic neuropathy is associated with burning or shooting pain in the feet, which may sometimes be difficult to differentiate from atypical ischaemic rest pain. Distinguishing features include bilateral symmetric distribution, cutaneous hypersensitivity, and failure to relieve by dependency of the foot. Reduced vibration sensation and reflexes are also seen in diabetic neuropathy.

Nerve root compression may sometimes result in continuous pain but it has a dermatomal distribution and is associated with backache.

68
Q

1 A 34-year-old female undergoing a contrast swallow is noted to have a smooth anterior indentation on the oesophagram. Which of the following is most likely to be responsible?

a Right-sided aortic arch

b Aortic aneurysm

c Aberrant right subclavian artery

d Aberrant left pulmonary artery

e Double aortic arch

A

1 Answer D: Aberrant left pulmonary artery

An aberrant right subclavian and aortic aneurysm cause posterior indentation.

A right-sided aortic arch causes a right lateral indentation

and a double arch is responsible for a `reverse S’, or impressions on both sides of the oesophagus.

69
Q

2 A 34-year-old female presented with hypertension and an ejection systolic murmur. Which finding is most likely to support a diagnosis of true coarctation compared to pseudocoarctation?

a Figure `3’ sign on angiogram

b An associated bicuspid aortic valve

C Rib notching

d High positioned aortic arch

e Dilatation of the distal aorta

A

2 Answer C: Rib notching

Rib notching is not seen in pseudocoarctation. This entity, once thought benign, is due to kinking of the aorta. Although there is no obstruction to flow, the abnormal anatomy can lead to aneurismal dilatation distal to abnormality.

70
Q

3 A 72-year-old man presented to his general practitioner with progressive dyspnoea. He has a history of hypertension and is a smoker. The chest radiograph demonstrates mild cardiomegaly and widening of the mediastinum at the level of the aortic arch. After referral to a cardiologist, a contrastenhanced computed tomography examination was performed which revealed a soft tissue density mass in contact with the aortic arch with a central pool of contrast at the same density as the aorta. The pre-contrast images showed a fine rim of calcification peripherally. What is the most likely diagnosis?

a Aortic dissection with mediastinal haematoma

b Bronchogenic carcinoma invading the mediastinum

C Bronchogenic cyst

d Lymphadenopathy

e Atherosclerotic aortic aneurysm

A

3 Answer E: Atherosclerotic aortic aneurysm

A haematoma from a dissection is less likely to have a calcified rim and be in contact with the aorta. Given the history, an aortic aneurysm is most likely.

71
Q

5 A 25-year-old female underwent a CT to investigate a history of progressive leg claudication, abdominal pain, night sweats and myalgia. Circumferential thickening of the thoracic and abdominal aortic wall with a stenosis in the thoracic aorta was seen and a magnetic resonance angiogram performed. On short tau inversion recovery (STIR) sequences there is high signal in the aortic wall. What is the most likely diagnosis?

a Behcet’s disease

b Acute lymphoblastic leukaemia

c- Polyarteritis nodosa

d Takayasu’s arteritis

e Giant cell aortitis

A

5 Answer D: Takayasu’s arteritis

The description is that of an arteritis, with oedema in the aortic wall on the STIR sequences. In a young adult, Takayasu’s is the commonest large-vessel vasculitis.

72
Q

6 A previously fit and well 45-year-old male presented with fever, abdominal pain and weight loss. Clinical examination was unremarkable. The erythrocyte sedimentation rate was raised but his leucocyte count was normal. After referral to the surgical team a CT of his abdomen was performed. This showed thickening of the wall of the ascending colon with pericolic fat stranding. Contrast was seen within the superior mesenteric vessel and there was a wedge-shaped area of low attenuation in the spleen. A selective angiogram of the superior mesenteric artery was performed, which demonstrated multiple aneurysms measuring between 1 and 5 mm. What is the most likely diagnosis?

a Ischaemic colitis

b Systemic lupus erythematosus

C Polyarteritis nodosa

d Rheumatoid vasculitis

e Wegener’s granulomatosis

A

6 Answer C: Polyarteritis nodosa

The angiogram findings are classic although not pathognomonic of PAN. With no history of SLE or rheumatoid it makes these less likely. Up to two-thirds of patients with PAN have bowel features of the disease.

73
Q

13 A 56-year-old male patient presented with headache and swelling of his face and neck. A chest radiograph revealed several parenchymal opacities of varying sizes and widening of the superior mediastinum. What is the most likely cause of this presentation?

a Superior vena cava obstruction (SVCO) secondary to metastatic renal cell carcinoma

b SVCO secondary to tuberculous lymphadenopathy

C SVCO secondary to metastatic bronchogenic carcinoma

d SVCO secondary to metastatic thyroid carcinoma

e SVCO secondary to primary pulmonary lymphoma

A

13 Answer C: SVCO secondary to metastatic bronchogenic carcinoma

Malignant lesions account for 80-90% of SVCO, and of these bronchogenic carcinoma accounts 50% of cases.

74
Q

17 A 43-year-old woman presents with fever, malaise, arthralgia and myalgia. She also complains of pain in her left arm after activity and on examination her left radial pulse is weak. Her ESR is elevated but the other blood tests are unremarkable. A chest radiograph demonstrates an undulating contour of the lateral margin of the descending aorta and CT angiography reveals multifocal areas of thickening and enhancement of the wall of the thoracic aorta and left subclavian artery. Non-occlusive thrombus is seen within the left subclavian artery. What is the most likely diagnosis?

a Polyarteritis nodosa

b Rheumatoid vasculitis

C Churg-Strauss syndrome

d Microscopic polyangiitis

e Takayasu’s disease

A

17 Answer E: Takayasu’s disease

Takayasu’s disease causes granulomatous inflammation of large arteries.

Polyarteritis nodosa affects mediumsized vessels.

The other options are causes of small vessel vasculitis.

75
Q

22 A 62-year-old gentleman presented with syncopal episodes and intermittent pain and paraesthesia in his right hand especially when exerting his right arm. An MRI demonstrated obstruction of his right subclavian artery. Where in the artery is the obstruction most likely to be located?

a First part of the artery, at its origin

b First part of the artery, just distal to the right vertebral artery

C Second part of the artery, just distal to the deep cervical artery

d Second part of the artery, just proximal to dorsal scapular artery

e just lateral to the lateral border of the scalenus anterior muscle

A

22 Answer A: First part of the artery, at its origin

In the subclavian steal syndrome, there is stenosis of the first part of the subclavian artery proximal to Origin of the vertebral artery.

The scalenus anterior muscle divides the subclavian artery into three parts.

76
Q

23 A 3 9-year-old gentleman with frequent respiratory tract infections and recent onset stridor was assessed with a CT of his thorax. A vessel was seen passing above the right main stem bronchus and coursing between the trachea and oesophagus. The trachea was deviated to the left and there was atelectasis in the right upper lobe. Which abnormal vessel is most likely to be present?

a Aberrant left pulmonary artery

b Aberrant right pulmonary artery

C Double aortic arch

d Aberrant right subclavian artery

e Aberrant left subclavian artery

A

23 Answer A: Aberrant left pulmonary artery

An aberrant left pulmonary artery arises from the right pulmonary artery and passes above the right main bronchus and between the trachea and oesophagus to reach the left lung.

The right main stem bronchus may be bowed anteriorly and the trachea deviated to the left. There is an anterior indentation of the oesophagogram.

Atelectasis and/or obstructive emphysema may be seen in the right (and/or left) upper lobe.

It is associated with stenosis of the trachea, patent ductus arteriosus and absence of the pars membranacea.

77
Q

25 At one week a neonate developed lower-extremity cyanosis. Their chest radiograph showed an enlarged heart with increased pulmonary vasculature. The aorta had a figure `3’ appearance. What is the most common associated abnormality?

a Bicuspid aortic valve

b Ventricular septal defect

C Atrial septal defect

d Cerebral berry aneurysms

e Mycotic aneurysms in the descending aorta

A

25 Answer A: Bicuspid aortic valve

In the infantile form of aortic coarctation, there is a long segment of narrowing in the aortic arch after the origin of the innominate artery.

Unlike the adult form, up to 50% have associated abnormalities. The most frequently associated abnormality is the bicuspid valve.

78
Q

Ped) 31 A one-year-old boy presents with a six-day history of fever. On examination he has bilateral cervical lymphadenopathy, injected fissured lips, a strawberry tongue and bilateral non-purulent conjunctivitis. Which of the following should be performed in the acute setting?

a Neck ultrasound

b Slit-lamp examination of the eyes

C Echocardiogram

d Lumbar puncture

e Abdominal ultrasound

A

31 Answer C: Echocardiogram

The infant has Kawasaki disease, a multisystem vasculitis with a predilection for the coronary arteries.

Fifteen to twenty-five per cent of patients have associated coronary artery aneurysms, most commonly of the left coronary artery.

An echocardiogram is advised at the time of diagnosis, at two weeks and six to eight weeks after diagnosis.

Contrast-enhanced CT of the chest and MR can also demonstrate the aneurysms.

79
Q

Ped) 55 A seven-year-old girl presents with dysphagia and recurrent chest infections. She undergoes a barium study of her oesophagus, which reveals a right-sided impression in the upper chest and a left-sided impression slightly lower down. What is the most likely explanation for these findings?

a Right arch with aberrant left subclavian

b Right arch without aberrant vessels

C Left arch with aberrant right subclavian

d Double aortic arch

e Innominate artery compression syndrome

A

55 Answer D: Double aortic arch

In double aortic arch, the right arch is usually higher than the left. The right arch passes posterior to the trachea and the left arch passes anterior to the trachea.

The right arch supplies the right common carotid artery and right subclavian artery.

The left arch supplies the left common carotid artery and left subclavian artery.

Double aortic arch is rarely associated with congenital heart disease.

A left arch with aberrant right subclavian is the most common arch anomaly and causes a posterior impression on the mid-oesophagus.

A right arch with aberrant left subclavian also causes a posterior impression on the mid-oesophagus.

Innominate artery compression syndrome causes anterior tracheal compression 2 cm above the carina.

An aberrant right pulmonary artery causes an anterior oesophageal impression.

80
Q

67 An elderly former smoker with worsening ischaemic symptoms in both legs underwent diagnostic lower limb angiography. Independent of the signs and symptoms of his disease, what is the most likely site of atherosclerotic disease in the lower limb?

a Iliac artery

b Common femoral artery

c Superficial femoral artery

d Popliteal artery

e Tibial artery

A

67 Answer C: Superficial femoral artery

Atherosclerotic disease typically has a symmetrical pattern and develops at points of turbulent flow (e.g. bifurcations). In the lower limb the commonly affected sites are: SFA > Iliac artery > Tibial artery > Popliteal artery > CFA.

81
Q

68 A 31-year-old man developed fever and haematuria. Physical examination revealed multiple subcutaneous nodules and blood tests that his ESR was raised. Following an abnormal renal ultrasound and CT he underwent renal angiography. Multiple small intrarenal aneurysms were demonstrated. What is the most likely diagnosis?

a Atherosclerosis

b Ehlers-Danlos syndrome

C Fibromuscular dysplasia

d Polyarteritis nodosa

e Von Hippel-Lindau disease

A

68 Answer D: Polyarteritis nodosa

Renal artery aneurysms can be broadly divided into extrarenal and intrarenal.

Extrarenal causes include atherosclerosis and fibromuscular dysplasia.

Intrarenal aneurysms are usually due to polyarteritis nodosa (PAN).

Pan is a rare necrotising vasculitis that affects small and medium-sized arteries of multiple organs.

The kidney and liver are most commonly affected.

Patients may also develop characteristic subcutaneous nodules.

82
Q

69 A 17-year-old student presented with an acutely ischaemic left calf and foot and on further questioning he described several months of claudication in his left calf which was worse during periods of prolonged standing. He also reluctantly admitted to a single episode of intravenous drug use through the antecubital fossa. Lower limb arteriography demonstrated stenosis and poststenotic dilatation of the popliteal artery, which was noted to be deviated medially within the popliteal fossa. What is the most likely diagnosis?

a Atherosclerosis

b Baker’s cyst

c Buerger’s disease

d Mycotic aneurysm

e Popliteal entrapment

A

69 Answer E: Popliteal entrapment

In popliteal entrapment the popliteal artery is displaced medially around the medial head of gastrocnemius. It is common in young athletes, and should be considered in leg ischaemia without trauma in this age group.

83
Q

GIT) A 46-year-old female underwent a barium swallow to investigate dysphagia. An indentation was seen on the posterior aspect of the mid thoracic oesophagus. What is the most likely explanation?

a Aberrant left subclavian artery

b Aberrant right subclavian artery

c Normal aortic indentation

d Oesophageal web

e Impression of the cricopharyngeus

A

Answer B: Aberrant right subclavian artery

The right subclavian artery normally arises from the brachiocephalic trunk bifurcation. An aberrant right subclavian is the last branch of the aortic arch distal to the left subclavian and indents the oesophagus posteriorly.

84
Q

1 A 46-year-old male who was in a high-speed road traffic accident presents acutely to the Emergency Department. He has severe chest pain radiating to his back and is haemodynamically unstable. What finding on an anterior posterior chest radiograph is most specific for acute thoracic aortic injury?

a Widening of the mediastinum

b Widened right paratracheal stripe

c Indistinct aortic arch contour

d Obscuration of the aortopulmonary window

e Right-sided haemothorax

A

1 Answer C: Indistinct aortic arch contour

85
Q

2 You are asked to review a chest radiograph following a pacemaker insertion. The leads have been placed via the left subclavian approach and pass down the left mediastinal border before forming a loop with the tip projected over the right ventricle. What is the most likely explanation?

a Partial anomalous pulmonary venous return

b An atrial septal defect

C A persistent left superior vena cava

d A ventricular septal defect

e Normal appearance, no abnormality

A

Answer C: A persistent left superior vena cava

A persistent left superior vena cava (SVC) courses along the left mediastinal border and enters the coronary sinus, which is usually dilated. There is more often than not a right SVC as well.

86
Q

6 A 64-year-old male smoker was seen by the vascular surgical team in the outpatient clinic with increasing unilateral lower limb claudication. A duplex ultrasound showed a stenosis within the left popliteal artery. What is the most likely cause?

a Popliteal artery entrapment syndrome

b Post-traumatic stenosis

c Cystic adventitial disease

d Atherosclerosis

e Emboli

A

6 Answer D: Atherosclerosis

Popliteal artery entrapment is a rare cause of occlusive disease and is classically seen in young men.

Atherosclerosis is a common condition and although it is unusual to affect the popliteal region with comparative sparing elsewhere, it would still be the most likely cause in an older smoker.

87
Q

13 A 65-year-old man with tearing chest pain radiating to his back was investigated with a contrast enhanced CT of his thorax which demonstrated an intimal flap separating the aortic lumen into two separate channels. The flap was seen to originate just distal to the origin of the left subclavian artery and to extend into the left common iliac artery. An aortic dissection is diagnosed. What is the appropriate Stanford classification?

a Stanford type 1

b Stanford type 2

C Stanford type 3

d Stanford type A

e Stanford type B

A

13 Answer E: Stanford type B

The Stanford classification of aortic dissection determines management of aortic dissection.

Type B dissections involve only the descending aorta and are usually treated non-surgically.

Type A dissections always involve the ascending aorta. They may also involve a variable portion of the arch and descending aorta.

Type A dissections are managed surgically.

88
Q

14 A 47-year-old lady presented with sudden onset right hemiparesis after lifting a heavy shopping bag. CT of her brain demonstrated two foci of low attenuation within the posterior frontal lobe and anterior frontal lobe adjacent to the interhemispheric fissure on the left which involved both white and grey matter. In addition she was also noted to have an erythematous, swollen left calf which was confirmed with a Doppler study to be due to thrombus within the superficial femoral vein. What is the next most appropriate radiological investigation?

a CT venography

b CT arterography

C Echocardiography

d Abdominal ultrasound

e MRI brain

A

14 Answer C: Echocardiography

In the presence of a deep-vein thrombosis (DVT) an embolic stroke raised through a right to left shunt should be investigated. Under certain circumstances, for example during temporary raised intrathoracic pressure when lifting a heavy shopping bag, in the presence of an atrial septal defect (ASD) or ventricular septal defect (VSD), emboli may cross from the venous to arterial circulation. Therefore, in this case the search for a cardiac septal defect (most likely to be an ASD) with echocardiography is indicated.

89
Q

16 A 57-year-old solicitor presented with a swollen left calf. She is on hormone replacement therapy and has a history of pulmonary embolism. What ultrasound feature is most in keeping with a diagnosis of deep vein thrombosis?

a Venous distension on Valsalva

b Increased flow within the superficial femoral artery on squeezing the calf

c Decreased flow in superficial veins and deep collaterals

d Phasic flow with respiration

e Loss of phasic flow on Valsalva

A

16 Answer E: Loss of phasic flow on Valsalva

Direct evidence of thrombus: Inability to compress the vein with transducer pressure, intraluminal echogenic thrombus (acute clot may be poorly echogenic), increased luminal diameter in acute thrombus, reduced diameter in chronic thrombus, vein wall thickening and absent flow in occlusive thrombus.

Indirect evidence of thrombus: Loss of phasic flow with respiration/Valsalva suggesting proximal venous obstruction, loss of venous distension on Valsalva, minimal increase in flow on squeezing calf, increased flow in superficial veins and deep collaterals.

90
Q

22 A 65-year-old gentleman presented with syncopal episodes and intermittent pain and paraesthesia in his left hand when he exerted his left upper limb. A Doppler ultrasound demonstrated a stenosis of the left subclavian artery. In which artery would flow reversal be most likely?

a Right common carotid artery

b Right vertebral artery

C Right axillary artery

d Left vertebral artery

e Left subclavian artery

A

22 Answer D: Left vertebral artery

In subclavian steal syndrome there is flow reversal in the ipsilateral vertebral artery at the expense of the cerebral circulation. This can be confirmed with colour Doppler imaging or MR. The condition is usually secondary to atherosclerosis and males are more commonly affected. The condition is three times as common on the left. Angioplasty yields good long-term results but some individuals require bypass surgery.

91
Q

Ped) 38 A CXR performed for cough in a 10-year-old does not show any abnormality of the lungs. It is reported as having a posterior indentation of the mid-oesophagus. Which of the following is the most likely cause of this appearance?

a Right aortic arch without aberrant vessels

b Right aortic arch with aberrant left subclavian artery

C Double aortic arch

d Left aortic arch with aberrant right subclavian artery

e Left aortic arch without aberrant vessels

A

38 Answer D: Left aortic arch with aberrant right subclavian artery

A left arch with aberrant right subclavian affects 0.5 % of the population and is usually asymptomatic.

92
Q

61 A 47-year-old shop assistant presented with a long history of allergic rhinitis and asthma. In the previous few weeks she had also experienced increasing dyspnoea, arthralgia and diarrhoea, which was occasionally blood stained. Blood analysis showed an elevated eosinophil count, mild renal impairment and positive pANCA antibody. There was patchy, non-segmental, bilateral airspace opacification on HRCT with small nodules. What is the most likely unifying diagnosis?

a Wegener’s granulomatosis

b Goodpasture’s disease

C Histoplasmosis

d Polyarteritis nodosa

e Churg-Strauss disease

A

61 Answer E: Churg-Strauss disease

Churg-Strauss disease is a multi-system disease and is a variant of polyarteritis nodosa. It is characterised by rhinitis, asthma, peripheral blood eosinophilia and a systemic small-vessel granulomatous vasculitis.

Clinical manifestations are caused by a necrotising vasculitis, eosinophilic tissue invasion and extravascular granulomatous eosinophilic abscesses.

The imaging findings within the lungs are those of pulmonary haemorrhage and eosinophilic pneumonia.

93
Q

69 A patient with severe hypertension refractory to maximal medical therapy had a renal angiogram. What angiographic features would suggest that percutaneous transluminal angioplasty (PTA) would be beneficial?

a Mid-renal artery stenosis

b Multiple intra-renal aneurysms

c Ostial stenosis

d Small diameter renal arteries

e String-of-beads appearance

A

69 Answer E: String-of-beads appearance

Fibromuscular dysplasia most commonly manifests as a `string-of-beads’ appearance in the renal artery and responds well to PTA.

Atherosclerotic renal artery disease is less amenable to PTA although mid renal artery stenoses respond better than ostial stenoses.

94
Q

Ped) 70 A 10-year-old boy is found to have an incidental murmur. His CXR demonstrates a dilated subclavian artery, a dilated descending thoracic aorta, and rib notching. What is the likely diagnosis?

a ASD

b VSD

c PDA

d Pulmonary artery stenosis

e Aortic coarctation

A

70 Answer E: Aortic coarctation

Children with aortic coarctation can present at any age with a murmur, headaches from hypertension or claudication from hypoperfusion.

On CXR the `three’ sign may be seen; that is, dilated subclavian artery, which is the upper convexity; narrow point at the coarctation; and post-stenotic dilatation, which is the lower convexity.

Rib notching is usually seen by six to eight years of age.

Aortic coarctation can also present with congestive heart failure secondary to critical stenosis.

95
Q

10 A 27-year-old male presented to the Emergency Department following a road traffic accident. He was haemodynamically unstable and complained of severe chest pain. His initial supine chest radiograph was normal and abdominal examination was also unremarkable. A CT thorax was requested to investigate possible aortic injury. What is the most likely finding on CT of an aortic transection?

a A contour deformity on the inner aortic wall at the level of the isthmus

b Increased density within the superior mediastinum

C Pericardial effusion

d Contrast extravasation from the aorta at the level of the diaphragmatic hiatus

e Occlusion of the subclavian artery

A

10 Answer A: A contour deformity on the inner aortic wall at the level of the isthmus

Aortic transection occurs most commonly (90%) at the isthmus, typically presenting as a contour deformity.

96
Q

@# 13 A 46-year-old Russian sailor presented with malaise and chest pain. In the course of the investigations he underwent CT scanning of his chest and abdomen. His VDRL and MHA-TP (for Treponerna pallidurn) tests are positive. What is the most likely finding on his CT?

a Saccular aneurysm of the ascending aorta with thin, dystrophic wall calcification

b Saccular aneurysm of the ascending aorta with interrupted calcification, para-aortic gas collection and adjacent reactive lymph node enlargement

C Fusiform aneurysm of the descending aorta with cresenteric mural thrombus. There is ectasia of the remainder of the aorta with heavy atherosclerosis

d Fusiform aneurysm of the abdominal aorta demonstrating mural thickening and extensive surrounding fibrosis

e Fusiform aneurysm of the descending aorta with an irregular wall and active extravasations of intravenous contrast

A

13 Answer A: Saccular aneurysm of the ascending aorta with thin, dystrophic wall calcification

This is the typical description of a syphilitic aneurysm. Options B, C, D and E are typical of a mycotic aneurysm, an atherosclerotic aneurysm, an inflammatory aneurysm and an actively leaking aneurysm respectively.

97
Q

16 A middle-aged gentleman complained of frequent throbbing headaches and cramping of his lower limbs after moderate exertion. He was investigated and a CT of his thorax showed a narrowing at the junction of the aortic arch and descending aorta. Rib notching was visible on his chest radiograph. Which ribs are likely to be most severely affected?

a Superior border first to sixth ribs

b Superior border second and third ribs

C Inferior border third and fourth ribs

d Superior border seventh to ninth ribs

e Inferior border sixth to eighth ribs

A

16 Answer C: Inferior border third and fourth ribs

In localised (adult type) coarctation there is usually a short segment of narrowing near the ligamentum arteriosum. These patients may present with headaches due to hypertension and lower limb claudication secondary to poor perfusion.

Rib notching develops due to enlarged and tortuous intercostal arteries.

It involves the inferior aspect of the central and lateral thirds of the posterior ribs and is most pronounced in ribs three and four.

The first two ribs are usually spared because their intercostal arteries originate from the subclavian arteries.

Rib notching is commonly bilateral but may be unilateral in the presence of an aberrant subclavian artery.

98
Q

Ped) 25 A two-year-old boy presents with a one-week history of fever, vomiting and conjunctivitis. On examination he is found to have a red tongue, a rash on both elbows and bilateral non-purulent conjunctivitis. Which of the following is the most likely diagnosis?

a Takayasu’s arteritis

b Polyarteritis nodosa

C Kawasaki syndrome

d Stevens Johnson syndrome

e Henoch-Schonlein purpura

A

25 Answer C: Kawasaki syndrome

Kawasaki syndrome is an idiopathic acute febrile multisystem vasculitis involving large, medium and small arteries with a predilection for the coronary arteries.

It typically occurs in children under five years old and is associated with a fever for greater than five days.

Further features include cervical lymphadenopathy and erythema of palms and soles with desquamation.

99
Q

GU) 51 A 35-year-old patient is undergoing investigation of renovascular hypertension with MR angiography. The distal part of the renal artery is abnormal and the appearance is like a `string of beads’. The patient is diagnosed with fibromuscular dysplasia (FMD). What is the most appropriate treatment?

a Conservative management: spontaneous regression is the norm

b Surgical excision of involved segment

c Angioplasty without stenting

d Angioplasty and stenting

e Intravascular high-intensity focused ultrasound

A

51 Answer C: Angioplasty without stenting

FMD is commonest in young women and typically involves the mid and distal artery (as opposed to atherosclerotic ostial stenosis), bilaterally in two-thirds.

Other aortic vessels are affected in 1-2 %.

The string of beads sign is caused by alternating areas of stenoses and aneurysms and is seen in the commonest form, medial fibroplasia.

Treatment is balloon angioplasty, without stenting.

100
Q

CNS) 39 A 56-year-old man was investigated for intermittent arm claudication and gait problems. A recent Doppler scan performed as part of an insurance check up had shown no disease in the carotids but did show flow reversal in the left vertebral artery. An angiogram was performed which revealed occlusive disease in the left subclavian artery origin with flow reversal in the ipsilateral vertebral artery on delayed images. What is the most appropriate treatment?

a Open endarterectomy

b Endovascular endarterectomy

C Surgical bypass with vein graft

d Balloon angioplasty

e Endovascular stent placement

A

39 Answer E: Endovascular stent placement

The condition described is subclavian steal syndrome, which is a term used to describe retrograde blood flow in the vertebral artery associated with proximal ipsilateral subclavian artery stenosis or occlusion.

Most patients remain asymptomatic, while a few develop neurological symptoms following use of the ipsilateral arm.

The left subclavian is three times more commonly involved than the right and atherosclerosis is the underlying cause.

Endovascular management is common with a technical success rate of 86-100% and primary stenting of the subclavian artery is the procedure of choice.

101
Q
  1. A 60-year-old man presents in the accident and emergency department with acute back pain and chest pain. The chest radiograph shows a widened superior mediastinum. CT shows an intimal flap in the ascending aorta with contrast fillingon either side of the flap. The arch and descending aorta appear normal. What is the most likely diagnosis?

(a) Aortic aneurysm

(b) Stanford type B dissection of aorta

(c) DeBakey type I dissection of aorta

(d) DeBakey type II dissection of aorta

(e) DeBakey type III dissection of aorta

A
  1. (d) DeBakey type II dissection of aorta

This type involves only the ascending thoracic aorta and is surgically repaired.

102
Q
  1. A 40-year-old man with a history of intravenous drug abuse presents with back pain. CT shows an infrarenal aortic aneurysm and left psoas abscess. What is the most likely finding on CT?

(a) Lobulated, saccular aneurysm

(b) Fusiform aneurysm

(c) Pseudoaneurysm

(d) Periaortic gas

(e) Extensive mural thrombus

A
  1. (a) Lobulated, saccular aneurysm

The patient is likely to have a mycotic aortic aneurysm. Mycotic aneurysms are commonly saccular and lobulated and less commonly fusiform. They may beassociated with psoas abscess, discitis or osteomyelitis.

103
Q
  1. Which of the following are correct about thoracic aorta: (T/F)

(a) Stanford type B dissection affects the ascending aorta.

(b) Penetrating aortic ulcers are frequently multiple.

(c) Mycotic aneurysms are usually fusiform in configuration.

(d) Penetrating aortic ulcers usually progress to dissection.

(e) Type A dissection is more common than Type B.

A

Answers:

(a) Not correct

(b) Correct

(c) Not correct

(d) Not correct

(e) Correct

Explanation:

Dissection affecting ascending aorta is classified as Stanford type A and accounts for 75% of aortic dissection. Acute type A is a surgical emergency to avoid fatal complications. Stanford type B dissection affects the descending aorta.

Mycotic aneurysms are usually saccular and may grow rapidly.

Penetrating atherosclerotic ulcers usually progress to aneurysmal dilatation.

104
Q

@# 20. Which of the following are correct regarding fibromuscular dysplasia (FMD): (T/F)

(a) Is more common in males.

(b) Usually affects the intimal layer.

(c) Renal artery FMD is bilateral in 5% of cases.

(d) Can occur in veins.

(e) May present with a transient ischaemic attack.

A

Answers:

(a) Not correct

(b) Not correct

(c) Not correct

(d) Correct

(e) Correct

Explanation:

FMD is more common in females and presents between 15 to 50 yrs.

Medial fibroplasia is more common form of FMD with characteristic ‘strings of beads’ appearance. Intimal fibroplasia occurs in 10% of cases and adventitial hyperplasia is the rarest form.

Renal artery FMD is bilateral in approximately 30% of cases.

105
Q
  1. Which of the following are correct regarding popliteal artery disease:- (T/F)

(a) The popliteal artery is superficial to the popliteal vein on ultrasound.

(b) Popliteal artery entrapment syndrome (PAES) is a recognized condition in athletes.

(c) Popliteal artery aneurysms are bilateral in 50-70% of cases.

(d) Popliteal artery occlusion is seen in 30-50% of patients with complete knee dislocation.

(e) Balloon mounted stents are usually preferred to self-expanding stents when treating popliteal artery disease.

A

Answers:

(a) Not correct

(b) Correct

(c) Correct

(d) Correct

(e) Not correct

Explanation:

Popliteal artery lies posterior to the femur and anterior to the vein, thus artery is deep to the vein when scanning the popliteal fossa with ultrasound.

Stent placement in popliteal artery is reserved for cases of failed PTA when limb viability is threatened. Self-expanding stents are preferred because of superficial location of artery and concerns about extrinsic compression.

106
Q
  1. Vascular anatomy of the liver: (T/F)

(a) The middle hepatic vein divides the liver into anatomical right and left lobes (Couinard classification).

(b) At microscopic level, centrilobular veins drain into the portal circulation.

(c) The portal vein bifurcation is intrahepatic in 90% of cases.

(d) The right hepatic artery arises solely from the superior mesenteric artery in 10-15% of individuals.

(e) In the fetus, the ducutus venosus joins the right portal vein to the inferior vena cava.

A

Answers:

(a) Correct

(b) Not correct

(c) Not correct

(d) Correct

(e) Not correct

Explanation:

The hepatic lobule is the basic histological unit. The triads of hepatic arterioles, portal venules and bile duct branches run at the edge of the lobule. Blood flows from periphery inwards via hepatic sinusoids and is drained bycentrilobular veins which in turn drain into hepatic veins.

The portal bifurcation is extrahepatic in 40% - 80% cases.

Left portal vein is critical to fetal circulation as it receives blood from the placenta via the left umbilical vein and delivers it across the liver to the IVC via ductus venosus.

107
Q
  1. Which of the following are correct regarding aortic dissection: (T/F)

(a) Type A dissection is usually treated surgically.

(b) Type A dissection involves the ascending aorta.

(c) The true lumen is usually smaller than the false lumen.

(d) Diagnosis is most commonly made by digital subtraction angiography.

(e) Treatment options for type B dissection include stent grafting and balloon fenestration.

A

Answers:

(a) Correct

(b) Correct

(c) Correct

(d) Not correct

(e) Correct

Explanation:

Diagnosis is mostly done by cross sectional imaging (CT or MRI) both of which have high specificity and sensitivityabove 90%.

108
Q
  1. Which of the following are correct regarding features of mycotic aneurysms include: (T/F)

(a) Gradual enhancement with contrast.

(b) Fusiform structure.

(c) Adjacent vertebral osteomyelitis.

(d) Adjacent reactive lymph node enlargement.

(e) Tuberculosis is the commonest infective organism.

A

Answers:

(a) Not correct

(b) Not correct

(c) Correct

(d) Correct

(e) Not correct

Explanations:

Mycotic aneurysms are saccular types showing rapid enhancement and most commonly associated with Staphylococcus aureus (IV drug abuse and subacute bacterial endocarditis).

109
Q
  1. Which of the following are correct regarding Buerger’s disease (thrombo-angitis obliterans): (T/F)

(a) Is associated with cigarette smoking in 90-95%.

(b) Initially affects the proximal vessels and progresses distally.

(c) More commonly affects the upper limb.

(d) Has multiple corkscrew-shaped collaterals on angiography.

(e) Has skip lesions as a recognised feature.

A

Answers:

(a) Correct

(b) Not correct

(c) Not correct

(d) Correct

(e) Correct

Explanations:

Buerger’s disease initially affects the distal vessels and progresses proximally affecting the lower limbs more commonly.

110
Q
  1. Which of the following are correct regarding aortic dissection: (T/F)

(a) The Stanford Classification Type B aortic dissection involves the ascending aorta.

(b) Aortic dissections involving the ascending aorta account for 60-70%.

(c) There is an increased risk in Ehlers-Danlos syndrome.

(d) Contrast-enhanced CT is more accurate than transoesophageal echocardiography at identifying aortic dissections.

(e) Displacement of calcification in the aortic knuckle by >10mm is a useful sign.

A

Answers:

(a) Not correct

(b) Correct

(c) Not correct

(d) Not correct

(e) Correct

Explanations:

Type A dissection involves the ascending aorta.

Ehlers-Danlos syndrome is associated with increased risk of aortic aneurysms and not dissection.

111
Q
  1. Which of the following are correct regarding renal artery stenosis: (T/F)

(a) There is an association with neurofibromatosis.

(b) Fibromuscular dysplasia causes stenosis of the proximal renal artery.

(c) There is elevation of the rennin levels on renal vein sampling of the affected kidney by 50%.

(d) Duplex ultrasound is the investigation of choice.

(e) On IVU, there is early appearance of contrast material in the affected kidney.

A

Answers:

(a) Correct

(b) Not correct

(c) Correct

(d) Not correct

(e) Not correct

Explanations:

In FMD stenosis is more common in mid and distal renal artery.

In atherosclerotic stenosis proximal artery is involved.

MRI is investigation of choice.

Ultrasound is inadequate in 50% cases.

On IVU, there is delay due to reduced glomerular filtration rate.

112
Q
  1. Which of the following are correct regarding Takayasu’s arteritis: (T/F)

(a) External carotid artery branches are most commonly affected.

(b) The mean interval between symptom onset and diagnosis is 2-4 months.

(c) It is a recognised cause of fusiform aortic aneurysms.

(d) Stenotic lesions are more commonly seen in the thoracic than abdominal aorta.

(e) Ultrasound of the proximal common carotid artery shows circumferential thickening of the vessel wall.

A

Answers:

(a) Not correct

(b) Not correct

(c) Correct

(d) Correct

(e) Correct

Explanation:

Takayasu’s arterits mainly affects main aortic branches and pulmonary arteries.

External carotid artery involvement is seen in temporal arteritis.

Internal between symptom onset and diagnosis is 8 yrs.

113
Q
  1. Which of the following are correct regarding features of polyarteritis nodosa (PAN) include: (T/F)

(a) Multiple aneurysms.

(b) Luminal irregularities.

(c) Involvement of small veins.

(d) Necrotising vasculitis involving the small and medium sized arteries.

(e) Involvement of kidneys in 70-80%.

A

Answers:

(a) Correct

(b) Correct

(c) Correct

(d) Correct

(e) Correct

Explanation:

PAN is a systemic inflammatory disease, commoner in males and presents around 5th to 7th decade of life. Main differentials include microscopic polyangitis and SLE.

114
Q
  1. Regarding aortic transection (traumatic aortic injury), which of the following are correct? (T/F)

(a) The descending aorta is rarely involved.

(b) The most common site is the ascending aorta.

(c) The chest radiograph is normal in 30% of cases at presentation.

(d) The ‘left apical cap’ sign is highly specific for aortic transection.

(e) Chronic false aneurysm develops in 5% of cases.

A

Answers:

(a) Correct

(b) Not correct

(c) Correct

(d) Not correct

(e) Correct

Explanations:

The most common site is aortic isthmus (95%).

The ascending aorta is involved in 1% of cases.

The ‘left apical cap’ sign refers to mediastinal hematoma with extrapleural extension of blood (only 15% of mediastinal hematomas are due to aortic tear).

115
Q

@# 51. Which of the following are correct regarding coarctation of the aorta? (T/F)

(a) Adult coarctation is commonly associated with cardiac anomalies.

(b) It is a rare cause of infantile heart failure.

(c) The ductus arteriosus usually remains patent in adult type coarctation.

(d) Rib notching is usually present by 1 yr of age.

(e) A short segment of narrowing of ascending aorta is seen in infantile coarctation.

A

Answers:

(a) Not correct

(b) Not correct

(c) Not correct

(d) Not correct

(e) Not correct

Explanation:

Adult coarctation is short narrowing at ligamentum arteriosum.

It is rarely associated with cardiac anomalies and the ductus arterosium is usually closed.

Aortic coarctation is second most common cause of infantile heart failure (most common is hypoplastic left heart).

Rib notching involves 3rd to 8th ribs and in 75% of cases seen in over 6 yrs of age.